Emergency Medicine Neurology

Lakukan tugas rumah & ujian kamu dengan baik sekarang menggunakan Quizwiz!

One Step Further Question: In the setting of a suspected traumatic lumbar puncture, what is the accepted ratio of WBCs to RBCs?

Answer: 1 WBC per 700 RBCs.

One Step Further Question: What is the duration of action of edrophonium?

Answer: 5-10 minutes; it is a short-acting acetylcholinesterase-blocking agent.

One Step Further Question: What percentage of children will have a recurrent febrile seizure?

Answer: 50% of children < 12 months and 30% of children > 12 months. Overall, the risk of epilepsy in children is ~1%; postfebrile seizure, this risk doubles to ~2%.

One Step Further Question: What is a rebound headache?

Answer: A chronic or nearly daily headache associated with frequent use of medication for acute head pain.

One Step Further Question: What is status migranosus?

Answer: A severe migraine headache that persists >72 hours. Associated symptoms are usually debilitating and hospitalization may be required.

One Step Further Question: What are the classic findings in transverse myelitis?

Answer: A transverse level of sensory impairment, paraplegia, and sphincter disturbance

One Step Further Question: What is Miller-Fischer syndrome?

Answer: A variant of GBS characterized by opthalmoplegia and ataxia.

One Step Further Question: What scoring system may help risk stratify patients with TIA?

Answer: ABCD2.

One Step Further Question: What medication is used as initial treatment for idiopathic intracranial hypertension?

Answer: Acetazolamide 250 mg twice daily.

One Step Further Question: What is the name for an episode of transient monocular blindness caused by acute retinal ischemia?

Answer: Amaurosis fugax.

One Step Further Question: What is the treatment of choice for a carotid artery dissection?

Answer: Anticoagulation (heparin followed by warfarin).

One Step Further Question: Why are MS patients at increased risk for urosepsis?

Answer: Autonomic dysfunction causes urinary retention with high residual volumes.

One Step Further Question: What is the clinical presentation of anterior cord syndrome?

Answer: Bilateral loss of pain and temperature with preservation of vibration/proprioception and motor weakness below level of cord involvement.

One Step Further Question: Blockade of which receptor helps to reduce the incidence of extrapyramidal effects?

Answer: Blockade of muscarinic acetylcholine receptors suppresses the emergence of extrapyramidal effects.

One Step Further Question: In patients with neuroleptic malignant syndrome, what medications can be given instead of dantrolene?

Answer: Bromocriptine and amantadine.

One Step Further Question: Which medication class is considered effective for preventing vasospasms following an intracerebral hemorrhage?

Answer: Calcium channel blockers, particularly nimodipine, are effective for preventing vasospasms.

One Step Further Question: What US state has the highest death due to cysticercosis?

Answer: California. Likely due to the large Mexican immigrant population.

One Step Further Question: What antecedent bacterial infection is associated with GBS?

Answer: Campylobacter jejuni infection.

One Step Further Question: What bacteria species is most commonly associated with the development of GBS?

Answer: Campylobacter jejuni is the most common bacteria species

One Step Further Question: What is the medical treatment for patients with trigeminal neuralgia?

Answer: Carbamazepine (primary) or gabapentin (secondary).

One Step Further Question: Vertical nystagmus is always associated with which type of vertigo?

Answer: Central vertigo; peripheral vertigo is associated with horizontal or horizontal rotary nystagmus.

One Step Further Question: What laboratory procedure must be performed to diagnose xanthochromia?

Answer: Centrifugation of the CSF sample.

One Step Further Question: What finding on lumbar puncture increases the suspicion of cryptococcal meningitis?

Answer: Clear CSF with a high opening pressure.

One Step Further Question: What is the role of corticosteroids in CNS abscess?

Answer: Corticosteroids should only be given to patients with cerebral edema related to the abscess.

One Step Further Question: What test in patients with GBS correlates with need for intubation?

Answer: Decreased forced vital capacity.

One Step Further Question: Is the Miller-Fisher variant of GBS associated with ascending or descending paralysis?

Answer: Descending.

One Step Further Question: In the US, what is a common cause of imbalance due to peripheral disease?

Answer: Diabetic distal peripheral polyneuropathy.

One Step Further Question: What medication can be given as an alternative therapy to benztropine for patients with dystonic reactions?

Answer: Diphenhydramine

One Step Further Question: What is the primary treatment for Lyme disease?

Answer: Doxycycline.

One Step Further Question: Which mosquito-transmitted viruses commonly cause encephalitis in the US?

Answer: Eastern and Western Equine, West Nile, St.Louis and LaCrosse viruses.

One Step Further Question: What is the treatment of cauda equina syndrome?

Answer: Emergent decompression (a surgical emergency).

One Step Further Question: What is the classic triad of bacterial meningitis?

Answer: Fever, altered mental status, nuchal rigidity.

One Step Further Question: What is the most common cardiac manifestation of Lyme disease?

Answer: First degree AV block.

One Step Further Question: What is an internuclear ophthalmoplegia?

Answer: Frequently seen in patients with multiple sclerosis, it is an interruption of the medial longitudinal fasciculus causing adductor weakness in the affected eye. As a result, patients are unable to perform medial gaze.

One Step Further Question: What are the cornerstone therapies in multiple sclerosis?

Answer: Glucocorticoids and plasma exchange.

One Step Further Question: What is the prognosis for acute compression radial mononeuropathy?

Answer: Good, with most patients recovering fully by six months.

One Step Further Question: Which anatomical regions of the brain does infectious encephalitis have a predilection for?

Answer: Gray matter of the temporal lobes and inferior frontal lobe.

One Step Further Question: What systems need to be monitored for side effects from carbamazepine?

Answer: Hematologic (CBC) and hepatic (liver function tests).

One Step Further Question: What upper extremity fracture often compromises the radial nerve?

Answer: Humeral shaft fracture.

One Step Further Question: Headaches associated with a brain tumor are classically worse during what time of day?

Answer: In the morning when intracranial pressure is highest after a night of recumbent positioning.

One Step Further Question: What is the most common etiologic agent of discitis?

Answer: Infection with Staphylococcus aureus.

One Step Further Question: What is a Marcus Gunn pupil?

Answer: It is a relative afferent pupillary defect that results in bilateral pupil constriction when shining a light in the unaffected eye but an absent pupil response (or a paradoxial dilation) when a light source is directed toward the affected eye.

One Step Further Question: What is the Hunt and Hess Clinical Grading Scale?

Answer: It is a scoring scale for cerebral aneurysms and subarachnoid hemorrhage. The scale ranges from 0 (unruptured aneurysm) to 5 (deep coma, decerebrate posturing).

One Step Further Question: How long does it take for delirium to fully resolve?

Answer: It may take weeks to months for delirium to fully resolve.

One Step Further Question: What spinous process level corresponds to the pelvic brim?

Answer: L4.

One Step Further Question: How does Lambert-Eaton syndrome clinically differ from myasthenia gravis?

Answer: Lambert-Eaton syndrome is associated with small cell lung cancer in about 50% of cases, and symptoms improve with repetitive use.

One Step Further Question: What is the rate of intracranial hemorrhage in patients given tPA for acute myocardial infarction?

Answer: Less than 1%.

One Step Further Question: Bilateral facial nerve paralysis should alert the provider to which potential diagnoses?

Answer: Lyme disease, infectious mononucleosis, HIV infection, botulism.

One Step Further Question: What is a common cause of bilateral peripheral facial nerve palsy?

Answer: Lyme disease.

One Step Further Question: What is the most common cause of bilateral Bell's palsy?

Answer: Lyme disease.

One Step Further Question: What imaging modality should be used to evaluate a patient for vertebrobasilar insufficiency?

Answer: MRI of the brain with MRA.

One Step Further Question: What are the most important infectious causes of seizures?

Answer: Meningitis, encephalitis, cerebral abscess, cerebral parasitosis and opportunistic infections associated with HIV.

One Step Further Question: What antibiotics should be used in a patient with meningitis and a confirmed cephalosporin allergy?

Answer: Meropenem or chloramphenicol can replace the third generation cephalosporin.

One Step Further Question: How are migraine headaches affected by pregnancy?

Answer: Migraines are vastly improved in most pregnant women.

One Step Further Question: What is Miller Fisher syndrome?

Answer: Miller Fisher syndrome is a rare form of GBS characterized by the triad of ophthalmoplegia, ataxia, and areflexia.

One Step Further Question: How will a patient with myasthenia gravis respond to edrophonium chloride testing if she is having a cholinergic crisis?

Answer: Muscle fasciculations, cholinergic symptoms, and respiratory depression will occur within a few minutes of the test dose.

One Step Further Question: The motor fibers of the trigeminal nerve inervate which muscles?

Answer: Muscles of mastication.

One Step Further Question: What symptoms suggest the presence of increased intracranial pressure (ICP) in subarachnoid hemorrhage?

Answer: Nausea, vomiting, confusion and weakness suggest increased ICP.

One Step Further Question: If conservative treatment fails, what medications are considered to treat persistent symptoms in patients with meralgia paresthetica ?

Answer: Neuropathic pain medications, such as gabapentin, phenytoin or carbamazepine.

One Step Further Question: What medication should be administered to help reduce the vasospasm associated with subarachnoid hemorrhage?

Answer: Nimodipine.

One Step Further Question: Do patients with alcohol withdrawal seizures require chronic anticonvulsant therapy?

Answer: No, medications such as phenytoin are neither necessary nor effective.

One Step Further Question: What role do neuromuscular blocking drugs have in refractory status epilepticus?

Answer: Nondepolarizing neuromuscular blocking drugs will stop the tonic-clonic movements, but will not affect abnormal neuronal firing. Patient will need an EEG.

One Step Further Question: Which arbovirus is treatable with existing antiviral medications?

Answer: None.

One Step Further Question: What are the 3 things that must be present to reliably exclude SAH?

Answer: Normal head CT scan, absence of xanthochromia, and zero or few RBCs in the CSF.

One Step Further Question: What is the characteristic finding on lumbar puncture for normal pressure hydrocephalus?

Answer: Normal opening pressure.

One Step Further Question: What common conditions are associated with carpal tunnel syndrome?

Answer: Obesity, diabetes, hypothyroidism, pregnancy, and renal failure.

One Step Further Question: Rifampin will turn urine what color?

Answer: Orange

One Step Further Question: What does the mini mental state examination test for?

Answer: Orientation, memory, attention.

One Step Further Question: What is the difference between plegia and paresis?

Answer: Plegia refers to paralysis (flaccid, no movement) while paresis refers to weakness (movement, but abnormal strength).

One Step Further Question: What are the CSF findings suggestive of MS?

Answer: Pleocytosis (50%), elevated gamma globulin (70%), oligoclonal bands (85%-95%).

One Step Further Question: What is the most common sequelae after lumbar puncture?

Answer: Post-LP headache which can be seen in up to 40% of patients.

One Step Further Question: What artery is typically involved in strokes causing homonymous hemianopsia?

Answer: Posterior cerebral artery.

One Step Further Question: What is a complication of tension headache?

Answer: Rebound headaches with frequent analgesic use.

One Step Further Question: What is a myasthenic crisis?

Answer: Respiratory failure leading to mechanical ventilation.

One Step Further Question: What is the pathognomonic finding for SAH on fundoscopic exam?

Answer: Retinal subhyaloid hemorrhage is seen in 11%-33% of cases.

One Step Further Question: What therapy should household contacts of patients with confirmed meningitis receive?

Answer: Rifampin (oral). Ciprofloxacin (oral) and ceftriaxone are alternatives.

One Step Further Question: What is the first line treatment for chemoprophylaxis for exposure to bacterial meningitis?

Answer: Rifampin with ciprofloxacin being an alternative.

One Step Further Question: What side effect is common with rifampin?

Answer: Secretions (tears, urine) will turn orange. Contact lens wearers should be warned of permanent staining

One Step Further Question: What is the treatment of idiopathic intracranial hypertension?

Answer: Serial lumbar punctures, acetazolamide and diuretics.

One Step Further Question: What is the most common bacterial cause of external shunt infection?

Answer: Staphylococcus epidermidis.

One Step Further Question: What is the treatment for a patient with a brain abscess and signs of increased intracranial pressure?

Answer: Surgical drainage.

One Step Further Question: Does surgery play a role in the management of trigeminal neuralgia?

Answer: Surgical options, including microvascular decompression and ablative procedures, may be considered in patients who are refractory to medical therapies.

One Step Further Question: Which common extrapyramidal symptom is irreversible?

Answer: Tardive dyskinesia develops over months to years of antipsychotic exposure and is usually irreversible.

One Step Further Question: What lab test is most likely to be abnormal in patients with spinal epidural abscess?

Answer: The erythrocyte sedimentation rate (ESR) is often elevated despite a frequently normal WBC count.

One Step Further Question: How long until the onset of action of phenobarbital?

Answer: The onset of action is 15-30 minutes.

One Step Further Question: What is the Phalen maneuver?

Answer: The opposing dorsal surfaces of the hands are pressed together with the wrists flexed for 60 seconds. It is positive for median nerve problems if this reproduces or worsens symptoms.

One Step Further Question: Which populations are at greatest risk for developing a ventriculoperitoneal shunt infection?

Answer: The very young, elderly, and those with a history of multiple shunt revisions.

One Step Further Question: Which segment of the spine is most commonly involved in transverse myelitis?

Answer: Thoracic. The cervical spine is rarely affected.

One Step Further Question: What is the other name of trigeminal neuralgia?

Answer: Tic douloureux.

One Step Further Question: What class of cluster headache abortive medications is contraindicated in patients with histories of ischemic heart disease or coronary vasospasm?

Answer: Triptan medications are contraindicated, as their vasoconstrictive mechanism of action may exacerbate these underlying heart diseases.

One Step Further Question: What is the most common long term complication of idiopathic intracranial hypertension?

Answer: Vision loss.

One Step Further Question: What finding on cerebrospinal fluid analysis is concerning for hemorrhage?

Answer: Xanthochromia.

3-year-old girl presents with a history of having a ventriculoperitoneal shunt placed at 22 months of age due to hydrocephalus. She has not required any revisions and has had no problems since its placement. According to Mom, for the past two days, the patient has been experiencing fever, headache, and fussiness. Which of the following is true about this patient's condition? CSF culture is often positive despite normal CSF analysis Most ventriculoperitoneal shunt infections occur after six months of placement Neisseria meningitidis is the most common infecting organism Urgent lumbar puncture is necessary

Correct Answer ( A ) Explanation: CSF shunt infections are classified as external (involving subcutaneous tissue around the shunt) or internal (involving the shunt and the CSF contained within the shunt). Due to the disruption of the blood-brain barrier, these patients are at increased risk of developing meningitis from typical pathogens (e.g., Streptococcus pneumoniae and Neisseria meningitidis). However, the most commonly cultured organism is Staphylococcus epidermidis, which accounts for approximately half of all infections. Many patients being evaluated for a shunt infection have a normal CSF analysis but a positive culture. Most shunt infections (B) present within the first two weeks of placement, with 80% occurring within 6 months. Patients with a ventriculoperitoneal shunt are at increased risk for developing meningitis (C) from typical pathogens, compared to the general population. However, the most common organism associated with ventriculoperitoneal shunt infection is Staphylococcus epidermidis. A lumbar puncture (D) has no meaningful role in the evaluation of suspected ventriculoperitoneal shunt infection. CSF analysis from a lumbar puncture often misses the diagnosis. Rather, it is critical that CSF is obtained from the shunt itself.

A 43-year-old man presents to the emergency department with a severe left-sided headache that started one hour ago, accompanied by stabbing unilateral periorbital pain. Physical exam reveals rhinorrhea, left-sided lacrimation, and redness of the eye. Administration of 100% oxygen provides modest relief. The headache could be best classified as which of the following types? Cluster headache Medication overuse headache Migraine headache Tension-type headache

Correct Answer ( A ) Explanation: Cluster headaches typically occur in middle-aged men, causing unilateral headaches with sharp periorbital pain. Lacrimation, rhinorrhea, redness of the affected eye, and restlessness commonly accompany the headache. A transient Horner syndrome with ptosis, miosis, and unilateral anhidrosis may be noted on physical exam. Diagnosis is clinical. Cluster headaches may be aborted with 100% oxygen via non-rebreather for 15 minutes. Acceptable medications for headache abortion include subcutaneous or intranasal sumatriptan, intramuscular or intravenous dihydroergotamine, or intranasal lidocaine. First-line preventative medications include oral lithium carbonate, verapamil, topiramate, or valproate. Medication overuse headaches (B) are usually associated with chronic daily headaches that become unresponsive to previously-effective doses of analgesic medications. Migraine headaches (C) may be lateralized or generalized, dull or throbbing, and are often associated with nausea, photophobia, phonophobia, or osmophobia. Though triptans may be effective for aborting migraine headaches, 100% oxygen would be unlikely to bring relief. Tension-type headaches (D) are typically generalized and described as "vice-like" or pericranial tightness. Focal neurological symptoms are not usually present. Effective treatments include over-the-counter analgesics and stress reduction, not 100% oxygen.

Which of the following patients is most likely suffering from dementia? A confused 65-year-old man brought from a house fire who exhibits good long-term recall, no recall of immediate events, normal vital signs, carboxyhemoglobin level of 7 A confused 65-year-old man brought from a house fire who exhibits poor long-term recall, no recall of immediate events, normal vital signs, carboxyhemoglobin level of 25 A confused 80-year-old man brought from home with fever, back pain, and urinary retention A confused 80-year-old man brought from home with fever, cough, and hypoxia A confused 80-year-old man brought from home with increased thirst, increased urination, and blood sugar monitor that reads "high"

Correct Answer ( A ) Explanation: Dementia results from a gradual loss of mental capacity with preservation of motor and speech. Dementia generally occurs in the elderly. Remote memories are often preserved. The most common types of dementia are Alzheimer's disease and vascular dementia, both insidious in onset. Symptoms may worsen acutely because of a concurrent medical condition. Physical exam and laboratory workup are generally unrevealing. The other patients described all have organic causes of delirium. Carbon monoxide poisoning (B) causes confusion, headache, vomiting, and syncope. A febrile illness (pyelonephritis (C) or pneumonia (D)​), or severe metabolic derangement (hyperosmolar hyperglycemic state (E)) can also lead to delirium in elderly patients. In the evaluation of all such patients, it is critical to differentiate acute delirium, which is a confusional state where functions of cognition and attention are impacted due to an underlying medical cause from chronic dementia

A 42-year-old man displays personality changes and confusion for 2 days. He denies pain. Upon presentation, you discover a weak right leg, speech impairment and low-grade fever. Upper extremity and left leg strength, rectal sensation and muscle tone, reflex testing and four-extremity sensory examination are normal. Nuchal rigidity, involuntary tremors, clonus and spasticity are absent. A brain MRI shows left temporal lobe edema. Cerebrospinal fluid analysis has an elevated number of red blood cells and the Gram stain is negative for bacteria. Which of the following is the most likely diagnosis? Encephalitis Meningitis Myelitis Sydenham's chorea

Correct Answer ( A ) Explanation: Encephalitis and meningitis manifest with similar symptoms, especially early in the infection. The classic meningitis triad is fever, headache and altered mental status, while altered mental status plus focal neurologic deficit is more descriptive of encephalitis. Herpes simplex encephalitis causes 10% of all diagnosed cases. More than half of these cases are fatal if untreated. HSV-1, commonly contracted in childhood (cold sores, fevers, and blisters), reactivates in adulthood to cause the majority of cases. Symptoms include up to 5 days of fever and headache, followed by behavior and personality changes, seizures, speech and memory deficits, hallucinations and altered consciousness. Frontal lobe pathology, leading to behavior and personality changes, and temporal lobe pathology, leading to memory and speech problems, are most common. Herpes encephalitis is associated with elevated red blood cells on CSF analysis. In adults, T2-weighted MRI reveals hyperintensity corresponding to edematous changes in the temporal lobes, inferior frontal lobes, and insula, with a predilection for the medial temporal lobes. Foci of hemorrhage occasionally can be observed on MRI. Untreated HSV encephalitis is associated with high morbidity and mortality. Therefore, in suspected cases of encephalitis without an obvious source, empiric treatment with intravenous acyclovir is recommended. Young children and the immunocompromised are at greatest risk for HSV disease. Meningitis (B) presents in a similar fashion to encephalitis early on. However, encephalitis is typically associated with behavioral and personality changes and neurologic deficits. Myelitis (C) (infection or inflammation of the spinal cord), typically presents with spinal pain, various sensory deficits, extremity weakness, reflex changes and bowel or bladder changes. Sydenham's chorea (D) occurs mainly in children with acute rheumatic fever or other Group A beta-hemolytic Streptococcus infection. It is characterized by face, hand and feet rapid jerking movements. Adult presentation is extremely rare.

Which of the following is true of Guillain-Barré syndrome? Elderly patients have a more severe clinical course It is associated with hyperreflexia Lumbar puncture will reveal low protein Rapid onset of GBS tends to have a benign recovery

Correct Answer ( A ) Explanation: Guillain-Barré syndrome (GBS) is an acute polyneuropathy characterized by immune-mediated peripheral nerve myelin sheath destruction. The classic GBS syndrome is preceded by a viral illness, followed by the subacute onset of ascending symmetric weakness or paralysis and loss of deep-tendon reflexes. The clinical course of GBS is more severe in the elderly. There is typically a more rapid recovery in children. GBS is associated with normal or diminished (B) deep-tendon reflexes. Lumbar puncture results show high protein (> 45 mg/dL) (C). Those patients with a rapid onset of symptoms (D) usually have a poorer prognosis.

Which of the following treatments is recommended as first-line management for patients with idiopathic intracranial hypertension? Acetazolamide Corticosteroids Serial lumbar punctures Ventricular Shunt

Correct Answer ( A ) Explanation: Idiopathic intracranial hypertension is caused by increased brain water content and decreased cerebrospinal fluid (CSF) outflow. Patients typically present with generalized headache that is gradual in onset. The headache may be worsened by eye movements. Additionally, any valsalva maneuver decreases cerebral venous return and will worsen the headache. Patients will often have visual complaints including transient vision loss secondary to ischemia of the visual pathways. Other typical symptoms include nausea, vomiting and lightheadedness. Fundoscopic examination reveals papilledema. Diagnosis is made based on an increased opening pressure found on lumbar puncture. Treatment focuses on decreasing CSF production initially with acetazolamide. Acetazolamide is a carbonic anhydrase inhibitor that can be used alone or in combination with furosemide. Corticosteroids (B) have shown some benefit but the mechanism of action is unknown. Serial lumbar punctures (C) to remove excess CSF is effective but invasive and painful. Ventricular shunt placement (D) and optic nerve sheath fenestration represent the last lines of therapy.

Which of the following is the average duration of a typical generalized tonic-clonic seizure? 1-2 minutes 10-15 seconds 30-40 seconds 4-5 minutes

Correct Answer ( A ) Explanation: Most generalized tonic-clonic seizures will stop on their own in one to two minutes. Consciousness returns gradually during the postictal period, but confusion and fatigue can last several hours. A seizure that lasts longer than five minutes or multiple seizures without a return to baseline mental status is defined as status epilepticus. Seizures typically are not this short (B and C), although the classic absence seizure seen in school-aged children may last just a few seconds. Some seizures last longer than two minutes (D). Those that approach five minutes should be considered status epilepticus.

Which of the following disease entities can be treated with the Epley maneuver? Benign paroxysmal positional vertigo Meniere's disease Vertebrobasilar insufficiency Vestibular neuritis

Correct Answer ( A ) Explanation: The Epley maneuver may be curative in patients with benign paroxysmal positional vertigo (BPPV). Vertigo is defined as the sensation or illusion of movement, often spinning, when the patient is standing still. There are numerous causes of vertigo. For matters of diagnosis and management, they are often divided into peripheral causes and central causes. Peripheral causes involve pathophysiologic changes outside of the central nervous system and are typically benign (with the exception of acoustic neuroma). Central vertigo, on the other hand, is generally caused by a dangerous etiology affecting the central nervous system (vertebrobasilar insufficiency, cerebellar hemorrhage and occlusion of the posterior inferior cerebellar artery). BPPV typically involves rapid onset of vertigo with nausea and vomiting that is elicited with moving the head to a certain position and relieved by moving the head away from that position. It is typically caused by the presence of an otolith in one of the semicircular canals in the inner ear. A Dix-Hallpike test can be performed to aid in diagnosis and an Epley maneuver can be used to dislodge the otolith from the semicircular canal leading to a resolution of symptoms. Menieres disease (B) is defined as longer episodes of vertigo with associated tinnitus and is not caused by an otolith. Vertebrobasilar insufficiency (C) is a central cause of vertigo and should be treated as a vascular ischemic episode. Vestibular neuritis (D) is caused by inflammation and does not respond to the Epley maneuver.

Which of the following patients in the setting of a known subarachnoid hemorrhage has the worst prognosis? A 39-year-old woman who is confused and has mild right lower extremity weakness A 55-year-old man with a severe headache and oculomotor nerve palsy A 65-year-old man with a mild headache, nausea, and an episode of vomiting A 70-year-old woman with headache, neck stiffness, and an episode of vomiting

Correct Answer ( A ) Explanation: The Hunt and Hess scale is one of several schemes used to classify the severity of a subarachnoid hemorrhage based on the patient's clinical condition at presentation. It is used as a predictor of outcome; higher grades correlate with lower survival rates. The patient who is confused with mild right lower extremity weakness is classified as Grade III and has a projected survival of 50%. A 55-year-old man with a severe headache and oculomotor nerve palsy (B) is a Grade II and has a 60% survival rate. A 65-year-old man with a mild headache, nausea, and an episode of vomiting (C) and a 70-year-old woman with headache, neck stiffness, and an episode of vomiting (D) are both Grade I and have a survival rate of 70%.

A 32-year-old man presents with acute onset right-sided facial weakness that he first noticed upon waking this morning. You ask him to smile as seen in the image above. Which of the following clinical features, if present, suggests a poorer prognosis? A vesicle found on the right tympanic membrane Dysgeusia Hyperacusis Unilateral symptoms

Correct Answer ( A ) Explanation: The patient has idiopathic facial nerve paralysis (Bell's palsy). Bell's palsy is characterized by rapid onset of unilateral facial paralysis. More than half of patients have a prodromal viral illness (rhinorrhea, myalgias, etc). Bell's palsy is associated with ear pain, decreased tearing, an overflow of tears onto the cheek (epiphora), hyperacute hearing (hyperacusis), and dysgeusia (an impairment of taste). Idiopathic facial nerve paralysis involves the lower motor neuron. Features that suggest an upper motor neuron cause include a slow progression of symptoms, recurrent symptoms, and intact forehead movement. A herpetiform vesicular eruption on the tympanic membrane, pinna, external auditory canal, soft palate, oral cavity, face, or neck suggests a diagnosis of Ramsay Hunt syndrome (herpes zoster oticus). In Ramsay Hunt syndrome, the pain is more severe than in Bell's palsy, and the prognosis is worse, with a lower incidence of complete facial recovery and the possibility of associated sensorineural hearing loss. Dysgeusia (B), hyperacusis (C), and unilateral symptoms (D) are not associated with a poorer outcome. Bilateral facial nerve paralysis suggests a diagnosis of Lyme disease, which is treated differently than Bell's palsy or Ramsay Hunt syndrome. Bell's palsy and Ramsay Hunt syndrome are treated with steroids and antivirals for 7-10 days.

A 34-year-old man presents with complaints of progressive lower extremity weakness over the past 2 days associated with 2 weeks of diarrhea that has since resolved. Vital signs are HR 89, BP 160/95, and RR 12. On exam, you note symmetric lower extremity weakness with intact sensation and absent ankle reflexes. Which of the following would help to support the most likely diagnosis? Albuminocytologic dissociation Improvement with steroids Increased forced vital capacity Selective enhancement of the dorsal root ganglion on MRI

Correct Answer ( A ) Explanation: The patient is most likely suffering from Guillain-Barré syndrome, a progressive, ascending demyelinating polyneuropathy that typically begins with lower extremity weakness. Classically, GBS presents with symmetric lower extremity weakness, decreased or absent deep tendon reflexes, and little or no sensory involvement (although variable). It follows a respiratory or gastrointestinal infection by weeks to days. Common organisms implicated include Campylobacter jejuni, cytomegalovirus, Epstein-Barr virus, and Mycoplasma pneumoniae. Elevated protein with only a mild pleocytosis on cerebrospinal fluid analysis (termed albuminocytologic dissociation) is highly specific in the clinical setting of suspected GBS. Patients with probable GBS should receive a neurology consult and be admitted for either intravenous immunoglobulin (IVIG) therapy or plasma exchange. Both IVIG and plasma exchange have been demonstrated to be more effective than placebo has, but no advantage is incurred upon administration of both therapies. Neither therapy has been proven more effective than the other. Selective enhancement of the anterior spinal nerve roots (D) on MRI is suggestive. Nearly 33% of patients diagnosed with GBS will require ventilator support. Factors associated with an increased need for ventilator support include a decreased forced vital capacity (FVC) (C) and a negative inspiratory force (NIF) less than 30 cm water. Other useful adjuncts include peak expiratory flow rate (PEFR) and arterial blood gas to evaluate for carbon dioxide retention. Corticosteroids alone (B) are no longer recommended, but IV steroids coadministered with IVIG may speed recovery. Regardless, steroids have no role in the diagnosis of GBS.

A 26-year-old woman presents to the ED with left-sided hemiplegia, right-sided pain and temperature sensation deficits. On your review of systems, she mentions an episode of unilateral painful vision loss that spontaneously regressed about two months prior. Which of the following is the most appropriate treatment for her condition? High-dose steroids Neurosurgical consultation Thrombolytics Triptan therapy

Correct Answer ( A ) Explanation: The patient is presenting with manifestations of multiple sclerosis (MS), a demyelinating disorder in which axonal myelin sheaths are damaged, leading to slowed nerve conduction. The mainstay of treatment during acute flares is high-dose methylprednisolone. MS can cause symptoms consistent with a Brown-Séquard syndrome, a circumstance clinically similar to hemisection of the spinal cord. Ipsilateral strength is decreased via the lateral corticospinal tract, and contralateral pain and temperature are decreased via the lateral spinothalamic tract, which decussates several levels distally. The patient's history of vision loss is suspicious for optic neuritis, making MS even more likely. There is no need for neurosurgical involvement (B) in this patient's case. This patient is not having an acute ischemic stroke, and her condition does not require thrombolytic therapy (C). Triptans (D) are used in the treatment of migraine headaches. Some complex migraines can present with vision loss, motor weakness, and paresthesias. Given that this patient has no history of migraines and is not complaining of a headache, it is unlikely that she has a migraine; triptan therapy is not indicated.

A 40-year-old man presents to the ED with the chief complaint of headache for two days. The headache is right frontal, constant, and severe. He has tried ibuprofen and acetaminophen without relief of the pain. He denies history of headaches, recent trauma, nausea, vomiting, and syncope. He has a past medical history of cervical disk protrusion for which he takes ibuprofen and gets manipulation by a chiropractor. His vital signs are T 36.6°C, BP 142/90, HR 82, RR 16, and oxygen saturation 99%. On examination, his right pupil is 3 mm and reactive; his left pupil is 6 mm and reactive. Extraocular movements are intact. He is unable to keep his right eyelid open against resistance. Visual acuity is 20/30 in both eyes with glasses. Fundoscopic exam is normal. His neck is supple. Strength is 5/5 in all extremities, gait is normal, and Romberg is negative. Which of the following is the most likely diagnosis? Carotid artery dissection Cluster headache Retinal artery occlusion Temporal arteritis Vertebral artery dissection

Correct Answer ( A ) Explanation: This patient has a carotid artery dissection. This is the most frequent cause of stroke in patients < 45 years old. Risk factors include minor neck trauma (cervical manipulation in this patient), family history of arterial disease, and connective tissue disorders. The most common presenting symptoms are unilateral neck pain or headache around the eye or frontal area. The pain is classically abrupt in onset. Carotid artery dissection is associated with a partial ipsilateral Horner's syndrome of miosis and ptosis without facial anhidrosis. Horner's syndrome results from a disruption of sympathetic innervation anywhere along the sympathetic chain. A carotid bruit may be heard on exam. The diagnostic study of choice is a MRI/MRA. If left untreated, it can lead to cerebral ischemia or retinal artery infarction and vision loss. Cluster Headaches (B) are typically unilateral and characterized by severe orbital, supraorbital, or temporal pain lasting 15-180 minutes, and recurring throughout the day. It is also associated with ipsilateral red conjunctiva, lacrimation, nasal congestion, and rhinorrhea. Seventy percent of the time these headaches resolve with oxygen administration. Based on presenting headache features, distinguishing vertebral from carotid artery dissection can be difficult. However, dissection of the vertebral artery (E) is typically associated with marked occipital or posterior neck pain and may be accompanied by focal neurological deficits, including ipsilateral facial numbness and contralateral pain and temperature sensory loss. Retinal artery occlusion (C) is painless and is associated with loss of vision. Fundoscopic exam reveals a pale retina with cherry red macula. Temporal arteritis (D) tends to occur in patients over 50 years of age and involves severe, throbbing, frontotemporal pain and jaw claudication. On exam, there is tenderness or decreased pulse in the temporal artery. ESR measurement is a useful screening test, but the final diagnosis is made by arterial biopsy.

A 20-year-old obese woman presents to the ED complaining of a headache that has progressively worsened over five weeks. The pain comes and goes and is occasionally associated with nausea and diplopia. She denies all other constitutional symptoms. She takes tetracycline for her acne. Vital signs are within normal limits. She has no focal neurologic deficits on physical examination. Which of the following findings is most consistent with the diagnosis she receives? Cerebrospinal fluid opening pressure on lumbar puncture of 35 cm H2O Conjunctival injection, lacrimation, nasal congestion, rhinorrhea, facial swelling, miosis, or ptosis seen on exam Elevated erythrocyte sedimentation of 90 mm/hr Fundoscopic exam that reveals unilateral optic disk swelling Symptoms that are relieved by the administration of high-flow O2

Correct Answer ( A ) Explanation: This patient has idiopathic intracranial hypertension, previously referred to as pseudotumor cerebri. This is a rare condition characterized by elevated intracranial pressure, headache, and papilledema. Although it typically occurs in obese women in late adolescence or early adulthood, it may also occur in men and children. The headache usually develops over weeks to months and may wax and wane. While there are often no neurologic findings other than papilledema, 6th nerve palsy and loss of peripheral vision may occur. Symptoms include headache, nausea, vomiting, subjective pulsatile intracranial "noise," and diplopia. It is linked to menstrual irregularities, pregnancy, use of oral contraceptives, weight gain, chronic steroid use, hypervitaminosis A, oral retinoids, and tetracycline administration. The diagnosis is made by lumbar puncture with measurement of opening pressure, which is universally elevated (> 25-30 cm H2O). CSF, however, should be otherwise normal. Treatment involves removal of CSF during lumbar puncture and decreased CSF production through administration of carbonic anhydrase inhibitors This constellation of signs is typically seen in cluster headaches (B), which are most common in young to middle-aged men and may be precipitated by alcohol and stress. Elevated erythrocyte sedimentation rate (C) is invariably present in patients with temporal arteritis, but the specificity of this finding is low. However, a normal sedimentation rate makes temporal arteritis extremely unlikely. This condition usually begins after age 55, is more common in women, and can present with jaw claudication, fever, malaise, weight loss, and anemia. Polymyalgia rheumatica (proximal motor weakness involving upper >> lower extremities) may also be present early in its course; however, idiopathic intracranial hypertension is associated with papilledema, which is usually bilateral. Unilateral papilledema is extremely rare (D) and can suggest ocular pathology such as an optic nerve glioma or optic neuritis. High-flow oxygen (E) is effective in reducing cluster headaches in approximately 70% of patients

A 36-year-old female with multiple sclerosis presents with progressive monocular vision loss over the preceding several hours. Which of the following exam findings would be expected? Afferent pupillary defect Diplopia on upward gaze Hazy cornea Sharp disc margins

Correct Answer ( A ) Explanation: This patient has optic neuritis, a common presenting symptom of multiple sclerosis (MS), which can also manifest in patients already diagnosed with MS. Optic neuritis is characterized by monocular progressive loss of vision over several hours to days caused by focal demyelination of the optic nerve. Vision loss, which occurs over several days and is usually unilateral, often is preceded by retrobulbar pain or extraocular muscle pain that may be reproduced with periorbital palpation. Optic neuritis may cause an afferent pupillary defect or Marcus Gunn pupil. This is found when a light moved from the unaffected eye to the affected eye results in pupil dilation instead of continued constriction. The optic disk may be pale. Although ocular pain most often resolves over several days, it may take months for the vision disturbances to resolve. Most patients at some time experience blurred vision, compromised color vision, or eye pain. Approximately 30% of patients with optic neuritis go on to develop MS within 5 years. Intravenous steroids have been shown to speed the recovery of vision loss when compared to placebo, but oral steroids have not and may be associated with an increase in optic neuritis occurrences.

A 30-year-old woman presents to the ED complaining of pain in the right side of her face for the last two days. The pain is sharp, severe, and lasts several seconds. The pain is associated with an involuntary movement of the side of her mouth. In between episodes, there is no pain. She has no past medical history. Her vitals include a temperature of 36.6°C, BP 122/78, HR 80, RR 20. Her exam is normal. Which of the following medications has been shown to relieve this condition and is the first line treatment? Carbamazepine 100 mg twice daily Gabapentin 100 mg three times daily Phenytoin 100 mg three times daily Prednisone 60 mg once daily

Correct Answer ( A ) Explanation: This patient has trigeminal neuralgia (tic douloureux), a disorder of the trigeminal nerve (cranial nerve V). The treatment is carbamazepine 100 mg twice daily. Carbamazepine is thought to work by reducing postsynaptic tetanic contractions. It is used to treat trigeminal neuralgia and is used as an antiepileptic. It is the first line treatment and the main antiepileptic used for this condition. Trigeminal neuralgia is characterized by paroxysms of severe unilateral pain in the trigeminal nerve distribution, sometimes described as recurrent bursts of an electric shock. The pain lasts only a few seconds and most commonly occurs on the right side of the face. On exam, you may be able to elicit pain by tapping trigger zones along the distribution of the trigeminal nerve. Otherwise, there should be no demonstrable physical findings. Gabapentin (B) is an analog of the neurotransmitter gamma-aminobutyric acid (GABA) and is used to treat seizures, postherpetic neuralgia, and restless leg syndrome. It is sometimes used as a second-line agent for trigeminal neuralgia. Phenytoin (C) is used to treat seizures. It is sometimes used as a second-line agent for trigeminal neuralgia. Prednisone (D) is an oral steroid. It is often used in the treatment of cranial nerve VII palsy, or Bell's palsy, but has no specific indication for trigeminal nerve disorders.

A 45-year-old woman complains of two days of fluctuating diplopia and dysphagia. Her exam reveals a left cranial nerve VI palsy, ptosis, and proximal muscle weakness in her extremities. She notes significant exercise intolerance over the past two days as well. What is the pathophysiology of her disease? Antibodies to the acetylcholine receptor at the neuromuscular junction Inhibition of acetylcholine release at the synapse Mutation of superoxide dismutase causing cell death Reduced number of dopamine receptors in the midbrain

Correct Answer ( A ) Explanation: This patient is having a myasthenic crisis, caused by antibodies to the acetylcholine receptor (AChR) at the neuromuscular junction. Failure to respond to acetylcholine stimulation leads to decreased muscle fiber activation and muscle weakness. The antibodies cause AChR degradation, dysfunction, and blockade. Ptosis and diplopia are most commonly the first symptoms. Patients may also experience dysphagia, proximal muscle weakness, and dyspnea. A full myasthenic crisis may lead to respiratory failure due to flaccid paralysis of the muscles of respiration Botulinum toxin inhibits acetylcholine release at the synapse (B). It is produced by Clostridium botulinum, an anaerobic spore-forming bacterium. C. botulinum occurs naturally in the soil and causes human disease when ingested. Botulism is associated with a descending symmetric paralysis. Amyotrophic lateral sclerosis (ALS) is caused by a mutation of superoxide dismutase (C) leading to an abnormal response to stress and cell death. Central and peripheral nerves are affected with sparing of sensory and cognitive function. Initial symptoms are muscle weakness, atrophy, fasciculations, dysarthria, and dysphagia. Parkinson's disease is associated with depigmentation and dopaminergic neuron loss (D) in the midbrain, especially the substantia nigra. It is also characterized by Lewy bodies, which are cytoplasmic inclusions seen under the microscope. Parkinson's disease is characterized by tremor, cogwheel rigidity, bradykinesia or akinesia, and postural and equilibrium impairment.

A 17-year-old boy experiences involuntary contractions of the facial and neck muscles after administration of haloperidol. The patient has arching of the back, deviation of the head to the left and protrusion of the tongue. What management is indicated? Benztropine Cyproheptadine Dantrolene Diazepam

Correct Answer ( A ) Explanation: This patient presents with a dystonic reaction after administration of a neuroleptic agent and should be given benztropine. Acute dystonia is the most common adverse effect seen after the administration of neuroleptic agents and occurs in up to 5% of patients. This reaction is caused by disruption in the balance between dopaminergic and cholinergic pathways in the basal ganglia. They tend to occur within 48 hours of administration of a neuroleptic agent. Patients often experience tongue protrusion, acute torticollis, sustained eye deviation (usually upwards) and arching of the back. Laryngospasm is rare but potentially life threatening. Because dystonic reactions occur as a result of cholinergic dominance, they should be treated with anticholinergic agents including benztropine and diphenhydramine. These agents typically reverse symptoms immediately.

A 44-year-old woman presents with involuntary movements of the neck, face and tongue. She reports that she was seen earlier in the Emergency Department for vomiting and received metoclopramide. What management is indicated? Benztropine intramuscular Lorazepam intramuscular MRI of the brain Noncontrast CT scan of the head

Correct Answer ( A ) Explanation: This patient presents with a dystonic reaction after the administration of metoclopramideand should be given benztropine. Acute dystonia is the most common adverse effect seen after the administration of neuroleptic agentsandcan occur in up to 5% of patients.It can also occurwith medications that antagonize central and peripheral dopamine receptors such as metoclopramide. This reaction is caused by disruption in the balance between dopaminergic and cholinergic pathways in the basal ganglia. They tend to occur within 48 hours of administration.Patients often experience tongue protrusion, acute torticollis, sustained eye deviation (usually upwards) and arching of the back. Laryngospasm is rare but potentially life threatening. Because dystonic reactions occur as a result of cholinergic dominance, they should be treated with anticholinergic agents including benztropine and diphenhydramine. These agents typically reverse symptoms immediately.

A 33-year-old woman presents with intermittent, intense shooting pain to the left side of the face. She states that the pain begins near her ear and radiates to her chin. The pain is often brought on by chewing and brushing her teeth. Which of the following managements is most likely indicated? Carbamazepine Dental X-rays Mandible CT scan Prednisone

Correct Answer ( A ) Explanation: This patient presents with symptoms consistent with trigeminal neuralgia and should be treated with carbamazepine. Trigeminal neuralgia is a syndrome characterized by sudden paroxysms of lancinating pain in one or more of the trigeminal nerve distributions. The syndrome manifests with unilateral facial pain triggered by chewing, brushing the teeth or touching the affected areas of the face. Hot and cold temperature exposure can provoke symptoms as well. The maxillary (V2) and mandibular (V3) branches of the trigeminal nerve are most commonly affected. The diagnosis is made based on clinical presentation. First line therapy for trigeminal neuralgia is with carbamazepine. Refractory cases may require surgical management. Dental X-rays (B) are useful in finding small apical abscesses of teeth but will not have utility in trigeminal neuralgia. A CT scan of the mandible (C) is most useful after acute trauma to diagnose mandible fractures. Trigeminal neuralgia does not result from inflammation so there is no role for prednisone (D) or other steroids.

A 42-year-old woman presents with severe facial pain. The patient reports she has had episodes of severe, shooting pain on the lower half of the left side of her face. Her neurologic examination is normal. Her medical history is positive for herpes zoster. Which of the following is the appropriate next step? Carbamazepine prescription CT scan of the head Prednisone prescription Referral for EEG

Correct Answer ( A ) Explanation: Trigeminal neuralgia is manifested by intermittent episodes of severe pain in the distribution of the fifth cranial nerve. The maxillary and mandibular branches of the trigeminal nerve are most commonly affected. Trigeminal neuralgia is thought to be idiopathic although there may be some component of vascular compression as this has been found on surgical exploration of the trigeminal nerve root. Patients will often describe physical triggers that lead to an episode of severe pain. These include: chewing, shaving, brushing the teeth or touching the involved area of the face. In the absence of any neurologic deficit, no imaging is necessary. The antiepileptic carbamazepine is the medical treatment of choice for this condition. Other medications that have been tried but not shown to have more effectiveness include gabapentin, phenytoin, valproic acid, baclofen, lamotrigine and levetiracetam. If medical therapy fails, surgical procedures aimed at the destruction of the trigeminal ganglion may be attempted. A CT scan of the head (B) is not indicated unless a neurologic deficit is identified. Structural lesions including tumors, vascular malformations and aneurysms could cause the neuralgia. Prednisone (C) is not part of the treatment regimen for trigeminal neuralgia. Prednisone is used as therapy in the treatment of Bell's palsy, which is a paralysis of the peripheral facial nerve. The underlying pathology of Bell's palsy is thought to be due to inflammation of the nerve and prednisone is effective at improving paralysis when compared to placebo. A referral for EEG (D) is not part of the work-up for trigeminal neuralgia. Although antiepileptic medication is used in the treatment of the syndrome, seizures themselves do not play a role in causing the neuralgia.

A 59-year-old woman with a history of poorly controlled hypertension presents to the emergency room for a sudden, very severe headache that began 50 minutes ago. She is disoriented and vomiting. A CT of her head showed no acute abnormalities. Which of the following is the next best step in her management? Electroencephalogram Lumbar puncture MRI head Supportive care

Correct Answer ( B ) Explanation: A lumbar puncture is the next best step to rule out a spontaneous subarachnoid hemorrhage (SAH). This patient has exhibited the classic symptoms of a SAH: a sudden headache, often of severity never experienced previously, nausea and vomiting, and impairment of consciousness that may progress to coma. A physical exam can show nuchal rigidity. Focal neurologic signs usually only present in the setting of a focal intracerebral hematoma. The patients history might reveal one or more risk factors for intracerebral hemorrhages, including recent head trauma, hypertension, tobacco use, or heavy alcohol use. Females of advancing age are at the greatest risk. At the first suspicion of a SAH, a CT scan, with CT angiography if possible, should be performed. If the CT scan does not confirm a hemorrhage, a lumbar puncture must be performed to fully rule out the diagnosis. An intracerebral hemorrhage is suspected when the cerebrospinal fluid is bloodstained in a quantity greater than that of a traumatic tap. Xanthochromia is also indicative of cerebral bleeding. Once diagnosed, patients need admission, and cerebral angiography should be done to determine the source of bleeding. Surgical clipping or endovascular treatment by interventional radiologists is necessary to definitively treat the hemorrhage. Following this, measures to prevent rehemorrhaging during the critical days following an intervention are necessary. These include bed rest with avoidance of physical exertion, and use of antiemetics, stool softeners, and antitussives as applicable. Seizure prophylaxis, usually with phenytoin, may also be used. An electroencephalogram (EEG) (A) will not allow for visualization of an acute hemorrhage. An EEG is generally more useful in making the diagnosis of seizures and in determining brain activity in patients with altered mental status, which are not acute concerns in this patient. An MRI of the head (C) is incorrect, as this imaging usually takes a significantly longer time than a CT scan. Additionally, CT scans are often more sensitive for diagnosing intracerebral hemorrhages within the first 24 hours than MRI scans. Supportive care (D) is important to consider if the diagnosis of an intracerebral hemorrhage, or equally critical condition, is definitively ruled out. At that point, clinicians may consider other, less life-threatening causes of head pain, such as migraine headaches, and provide support as applicable.

A 77-year-old woman presents with 2 days of bowel incontinence, leg pain, and difficulty ambulating. Lower extremity examination reveals bilateral sensory deficits of the lumbosacral dermatomes, bilateral hyporeflexia, negative Babinski testing, and weak rectal tone. There is no lower extremity spasticity. Cranial nerve and upper extremity examination is normal. These symptoms are most likely the result of injury to which of the following structures? Lumbar disk Lumbosacral nerve roots Spinal cord gray matter Spinal cord white matter

Correct Answer ( B ) Explanation: A normal adult spinal cord ends as the conus medullaris at the L1-L2 level. The remaining L2-S4 spinal nerve roots that occupy the subdural space from the conus to the filum terminale are called the cauda equina. The cauda equina is a collection of post-ganglionic nerves (lumbosacral nerve roots), so injury to it presents as a lower motor neuron injury pattern with flaccid tone, paresis or paralysis, and hyporeflexia (or areflexia). Upper motor neuron symptoms such as hyperreflexia and clonus, spasticity and upgoing toes (positive Babinski test) are absent. The clinical presentation of cauda equina syndrome includes back pain, perianal numbness, loss of rectal tone, bowel and bladder (urinary retention followed by incontinence) abnormalities, leg weakness, loss of bulbocavernosus reflex, and loss of deep tendon reflexes. Cauda equina syndrome most commonly results from a herniated disk, but can also occur from trauma, an infection (epidural abscess), or a hematoma (epidural hematoma). A lumbar disk abnormality (A), such as herniation, typically causes unilateral radicular pain and symptoms, but it is almost exclusively associated with normal rectal tone and perineal sensation. Spinal cord tumor, or other spinal cord neuronal cell body disease affecting the gray matter (C), would cause upper motor neuron symptoms. Hyperreflexia, positive Babinski testing and spasticity are not present in this patient. Spinal cord neuronal axon or tract disease affecting the white matter (D), as in multiple sclerosis, usually presents earlier in age, and with upper extremity and cranial nerve neurologic findings, both of which are absent in this case.

Which of the following characteristics helps to differentiate delirium from dementia? Attention is spared in delirium Delirium has an acute onset Delirium has no change in level of consciousness PET scanning is abnormal in delirium

Correct Answer ( B ) Explanation: An important distinction between delirium and dementia is the time of onset. Delirium is characterized by its rapid onset and fluctuating course. It leads to acute changes in mental status, marked by inattention, poor concentration, and fluctuating levels of consciousness. It commonly occurs in hospitalized elderly patients. Many disorders can have similar clinical features of delirium and patients with delirium are often mistakenly diagnosed with dementia or depression. Delirium, dementia, and depression can coexist within the same patient. When an acute change in mental status occurs, however, the diagnosis of delirium should be presumed until proven otherwise. Treatment involves identifying and treating the underlying causes. Clinicians should minimize the use of physical restraints as much as possible and encourage consistent presence of family or caregivers familiar to the patient. Inattention (A) is a distinguishing feature between delirium and dementia. Attention is spared in early and moderate dementia, whereas it is always impaired in delirium. Altered levels of consciousness (C) should not be present in mild or moderate dementia or depression, however it is seen in delirium. PET scanning (D) cannot differentiate acute delirium from dementia.

A 22-year-old man presents to the ED with unilateral throbbing headache. He denies family history of headache, however, he has had this type of headache before. Beginning around age 20 years, he started having four to six "flares" a day for six weeks at a time. Each flare would last 30 minutes and they mainly occurred at night. Eventually they resolved, but reappeared after his 21st birthday and lasted for about two months. He had been asymptomatic until two days ago when the headaches returned. Physical examination shows conjunctival injection and rhinorrhea. Which of the following treatments should most likely be initiated first in this patient? Acetaminophen, aspirin, and caffeine Oxygen Topiramate Verapamil

Correct Answer ( B ) Explanation: Cluster headaches occur most often in middle-aged men. It is described as a recurrent, unilateral, excruciating periorbital headache that lasts from 15 minutes to three hours. General headaches are typically non-pulsatile and constant, frequently occurring at night. Cluster headaches on the other hand are associated with ipsilateral conjunctival injection, lacrimation, and nasal congestion. Cluster headaches classically take place in groups over days to weeks, occurring at the same time of day and in the same location. Acute attacks can be treated with 100% oxygen. Other treatments include triptans, ergotamines, intranasal lidocaine, and butorphanol.

A 23-year old man presents to the emergency department with a right-sided headache lasting 30 minutes. He reports having these headaches 3-5 times a day with associated rhinorrhea and lacrimation. Which of the following is the initial abortive treatment of choice? Naproxen Oxygen Prednisone Verapamil

Correct Answer ( B ) Explanation: Cluster headaches occur most often in middle-aged men. They are described as recurrent, unilateral, excruciating periorbital headache that lasts from 15 minutes to 3 hours. Headaches are nonpulsatile and constant and frequently occur at night. They are associated with ipsilateral autonomic symptoms such as eyelid edema, nasal congestion, lacrimation, and forehead sweating. Horner's syndrome may also be present. Cluster headaches classically take place in groups over days to weeks, occurring at the same time of day and in the same location. The mainstay of treatment is 100% oxygen with tripans used as an adjunct for abortive therapy. Other abortive therapies include intranasal lidocaine and butorphanol. Cluster headaches are associated with an increased suicide rate due to the associated severity of pain. Verapamil (D), a calcium channel blocker, is used for prophylactic therapy, not abortive therapy. Naproxen (A) has no role in the treatment of cluster headaches. Prednisone (C) burst and taper is used to prevent acute recurrences. Other aspects of prophylaxis include the avoidance of triggers such as alcohol, stress, and medications causing vasodilation

A 35-year-old woman with a known history of seizure disorder is actively seizing in the ED. Which of the following is the first-line medication and route to treat her seizure? Intramuscular fosphenytoin Intravenous midazolam Oral lorazepam Rectal diazepam

Correct Answer ( B ) Explanation: In an actively seizing patient, attention is always directed to the airway first. A patient having a generalized tonic-clonic seizure has a suppressed gag reflex and is prone to aspiration of gastric contents. Therefore, patients should be placed in the left lateral decubitus position. First-line pharmacologic management for an actively seizing patient is a parenteral benzodiazepine. Benzodiazepines directly enhance GABA-mediated neuronal inhibition, affect clinical and electrical manifestations of seizures, and are highly effective at terminating seizure activity. Benzodiazepines have been shown to be more effective than phenytoin at terminating status epilepticus. They are as effective as parenteral phenobarbital, but phenobarbital is associated with a higher risk of hypoventilation and hypotension. The intravenous route is the preferred route to administer an antiepileptic because this has the quickest onset of action. Therefore, intravenous midazolam is the preferred agent from the choices listed above. Midazolam's onset of action is within 1 minute. In addition to the intravenous preparation, it is available in both intranasal and buccal formulations. It also has the least cardiovascular effects among benzodiazepines. Fosphenytoin (A) is a water-soluble prodrug form of phenytoin. Unlike phenytoin, fosphenytoin can be administered intramuscularly. Although fosphenytoin can be infused more rapidly than phenytoin can, the time to a therapeutic concentration of the active drug is the same as for intravenous phenytoin. Lorazepam (C) is considered a first-line agent for an actively seizing patient and is the most popular agent used. However, oral medications are contraindicated in an actively seizing patient due to the prolonged onset of action and risk of aspiration. Diazepam (D) is also a first-line agent for an actively seizing patient and is most commonly used as a rectal preparation in the pediatric population where intravenous access is not readily available.

A 32-year-old pregnant woman complains of burning pain on her right thigh. Neurological examination reveals hyperalgesic light touch sensation in the skin of the right, proximal lateral thigh. Further examination reveals normal right leg strength, normal distal leg sensation and a 2+ ankle reflex. Which of the following neuropathies is the most likely diagnosis? Femoral neuropathy Meralgia paresthetica S1 radiculopathy Tarsal tunnel syndrome

Correct Answer ( B ) Explanation: Meralgia paresthetica is the clinical syndrome of pain or dysesthesias, or both in the anterolateral, proximal thigh, due to a compressive neuropathy of the lateral femoral cutaneous nerve. Entrapment of this nerve under the inguinal ligament is common. Mean age of onset is 50-years-old, occurring equally in men and women. It is more common in patients with diabetes mellitus, obesity (higher Body Mass Index) and pregnancy, or those who have inguinal scar tissue or wear tight belts around the waist. Diagnosis is based on the above pain description, sensory abnormalities in the nerve's distribution and absence of abnormal lower leg neurologic findings. Electrodiagnostic findings are variable, as it is difficult to perform nerve conduction studies in obese individuals. Relief of pain with local injection of anesthetic confirms the diagnosis. Spontaneous remission is common, and 90% of patients respond to conservative treatment, which includes: education on weight loss and avoiding tight belts/garments and local steroid injection. Surgery is rarely necessary. Femoral neuropathy (A) could give rise to anterolateral thigh numbness, and a gravid pelvis could compress the lumbosacral plexus and proximal femoral nerve. However, other symptoms would be present, such as numbness in the anterior thigh and medial lower leg, decreased knee reflex and quadricep weakness. S1 radiculopathy (C) presents with back and leg pain, decreased ankle reflex, weak plantar flexion and numbness in the posterior leg, lateral ankle and sole. Tarsal tunnel syndrome (D) is due to entrapment neuropathy of the posterior tibial nerve under the transverse tarsal ligament at the medial malleolus. Symptoms include aching, numbness and burning dysesthesias on the sole, distal foot and toes.

A 23-year-old woman complains of a headache for 24 hours. She has no history of headaches, but they run in her family. Which of the following headaches is most likely to be a migraine? A fluctuating headache with seizure activity A pulsatile unilateral headache with nausea A sudden onset headache that is worse at onset An orbital headache with lacrimation

Correct Answer ( B ) Explanation: Migraine headaches can be remembered by the mnemonic POUND: Pulsatile, 4-72 hOurs, Unilateral, Nausea or vomiting, Disabling. Presence of at least four of these criterion has a likelihood ratio of 24 for a migrainous etiology of headache. Migraines are thought to be vascular in origin. They are treated with a variety of medications, including dihydroergotamine (DHE), triptans, metoclopramide, chlorpromazine, and ketorolac. Placement of the patient in a dark, quiet room and administration of IV fluids can also be helpful. Migraines can present with neurologic symptoms such as visual disturbances (scotoma), paresthesias, and auras, which can make the diagnosis difficult. In those patients without a history of headaches, more serious and life-threatening causes must be ruled out before a diagnosis of migraine is firmly established. A fluctuating headache with seizure activity (A) should prompt the clinician to consider central venous thrombosis (CVT), a venous occlusion from hypercoagulable states. CVT presents with headache, vomiting, seizures, and fluctuating neurologic symptoms. It is diagnosed by MRV. Cluster headaches (D) present with unilateral, severe, orbital, supraorbital, or temporal pain lasting 15-180 minutes. Patients often have ipsilateral red conjunctiva, lacrimation, nasal congestion, rhinorrhea, miosis, and/or ptosis. The patient is usually pacing and restless. High-flow oxygen is effective in up to 70% of patients. A sudden onset headache that is worst at onset (C) is concerning for subarachnoid hemorrhage (SAH), which is typically maximal at onset, sudden, and different from prior headaches. SAH is an emergent condition because of the many complications, including rebleed, infarction, and intracranial hypertension.

An 85-year-old man presents to the ED with gait disturbance and memory loss over a period of months. He finally came to the hospital when he developed urinary incontinence. What syndrome does this constellation of symptoms describe? AAlzheimer's disease BNormal pressure hydrocephalus CPsychiatric psychosis DWernicke-Korsakoff syndrome

Correct Answer ( B ) Explanation: Normal pressure hydrocephalus (NPH) is a potentially reversible cause of dementia and is characterized by the triad of gait disturbance, memory loss, and incontinence. NPH results from CSF buildup in the ventricles leading to increased intracranial pressure with edema of the periventricular white matter and corona radiata. The periventricular white matter contains the sacral motor nerve fibers that produce gait instability; incontinence ensues when compressed. Alzheimer's disease (A) presents with progressive dementia but does not include motor abnormalities. It is important to remember, however, that many elderly patients have more than one diagnosis at the time of presentation. Psychiatric psychosis (C) is a functional cause of confusional state. Patients demonstrate a loss of contact with reality and may have auditory hallucinations, but consciousness is generally not affected. Further, there are no motor deficits associated with this condition. Wernicke-Korsakoff syndrome (D) is a manifestation of thiamine (vitamin B1) deficiency. This is usually secondary to alcohol abuse and manifests with the triad of ophthalmoplegia, ataxia, and impaired memory. In contrast to NPH, urinary incontinence or frequency are absent. The treatment of Wernicke-Korsakoff syndrome is thiamine.

Olanzapine is a commonly used medication in patients with schizophrenia. Which of the following statements is true regarding the rapid dispersing formulation (wafer) of olanzapine? At overdose levels it will cause bradycardia It is considered to be bioequivalent to olanzapine tablet Rarely associated with orthostatic hypotension The risk of extrapyramidal side effects is higher than with haloperidol

Correct Answer ( B ) Explanation: Olanzapine is an atypical antipsychotic used in the treatment of schizophrenia, bipolar disorder, and acute agitation. It affects multiple receptors (serotonin, dopamine, muscarinic, adrenergic, histamine), but has the greatest affinity as a blocker of the serotonin (5HT2A) receptor and dopamine (D2) receptor. It is available in oral, rapid dispersible (wafer), and intramuscular preparations. In the ED, the rapid dispersible preparation is particularly valuable to manage acutely agitated patients due to its rapid absorption through the oral mucosa and resulting in rapid somnolence. Both oral and rapid dispersible (wafer) preparations are considered bioequivalent. Overdose leads to tachycardia, rather than bradycardia (A) due mainly to muscarinic blockade. Olanzapine is associated with a lower incidence of extrapyramidal effects (D) than haloperidol. Olanzapine is associated with orthostatic hypotension (C) due to the adrenergic (alpha) receptor blockade.

Which of the following lumbar puncture results would you expect to find with the head CT seen above? Tube 1: WBC 5, RBC 4000; Tube 2: protein 70, glucose 60; Tube 3: RBC 50 Tube 1: WBC 7, RBC 4000; Tube 2: protein 75, glucose 60; Tube 3: RBC 3200 Tube 1: WBC 80, RBC 3; Tube 2: protein 60, glucose 60; Tube 3: RBC 0 Tube 1: WBC 900, RBC 10; Tube 2 protein 200, glucose 15; Tube 3: RBC 1

Correct Answer ( B ) Explanation: The head CT scan shows a subarachnoid hemorrhage (SAH). Most SAHs are caused by a ruptured berry aneurysm in the circle of Willis. CT scan is the initial imaging modality to detect SAH and may (as with this example) show accumulation of blood in the subarachnoid space (yellow arrows). However, if the CT scan is negative, a lumbar puncture (LP) should be performed with comparison of red blood cell count in the cerebrospinal fluid (CSF) across tubes to differentiate SAH from a traumatic lumbar puncture. If the 1st and 4th tubes both have a significant number of RBCs, an SAH is most likely. A very large difference in RBC numbers (tube 1 >> tube 4) indicates a traumatic tap. No definitive number of RBCs needs to be present in the CSF to be diagnostic of SAH. Though not specifically addressed in this question, SAH can also be detected by identification of xanthochromia, a yellow appearance of the CSF due to blood breakdown and release of bilirubin. Xanthochromia may take up to 12 hours to develop and remain present for several weeks. Xanthochromia can be detected by direct visualization or, preferably, via spectrophotometry. Tube 1: WBC 5, RBC 4000; Tube 2: protein 70, glucose 60; Tube 3: RBC 50 (A) is most likely due to a traumatic LP. There is a very large difference in the number of RBCs from tube 1 to tube 4. Tube 1: WBC 80, RBC 3; Tube 2: protein 60, glucose 60; Tube 3: RBC 0 (C) is CSF indicative of aseptic meningitis, with a high number of WBCs, normal protein, and normal glucose. Tube 1: WBC 900, RBC 10; Tube 2 protein 200, glucose 15; Tube 3: RBC 1 (D) is CSF indicative of bacterial meningitis. There are a large number of WBCs, few RBCs, high protein, and low glucose.

A 12-year-old boy presents to the ED with headache, vomiting, and lethargy for two days. He had a ventriculoperitoneal shunt placed for hydrocephalus one year ago. What is the most likely cause of his shunt malfunction? Abdominal pseudocyst formation Choroid plexus obstruction Loculation Mechanical failure Slit ventricle syndrome

Correct Answer ( B ) Explanation: The most common cause of ventriculoperitoneal shunt malfunction is proximal tubing obstruction due to choroid plexus or increased protein within the CSF. Symptoms are consistent with increased intracranial pressure, leading to vomiting and irritability with a bulging fontanelle in infants, and headache, nausea, vomiting, lethargy, ataxia, and cranial nerve palsies in older children and adults. Treatment requires neurosurgical consultation with tapping of the shunt and measurement of the opening pressure. This will help ascertain whether the obstruction is proximal or distal and direct the ultimate intervention needed to fix the problem.

A 24-year-old man with a history of AIDS presents to the ED with fever and headache. His head CT scan is unremarkable. Cerebrospinal fluid results from lumbar puncture shows: WBC: 300/mm3 RBC: 3/mm3 Glucose: 35 mg/dL Protein: 209 mg/dL Gram stain: negative India ink: positive Which of the following is the most appropriate treatment at this time? Acyclovir Amphotericin B + flucytosine Fluconazole Itraconazole

Correct Answer ( B ) Explanation: This patient has fungal meningitis, most likely due to Cryptococcus neoformans, which is an opportunistic infection in AIDS patients. The most common presenting signs are fever and headache, followed by nausea, altered mental status, and focal neurologic deficits. Meningismus is uncommon. India ink staining of the cerebrospinal fluid (CSF) has a sensitivity of 60%-80%. Identifying cryptococcal antigen in the CSF has a sensitivity and specificity near 100%. Amphotericin B is the cornerstone of antifungal therapy for cryptococcal meningitis. The addition of flucytosine has been shown to enhance the mycologic and clinical effectiveness. Acyclovir (A) is used when viral etiologies of meningitis or encephalitis are suspected. Once patients have received a 14-day course of amphotericin and flucytosine (but not before then), they are started on fluconazole (C) for eight weeks to clear the CSF of pathogen. Some patients require lifelong maintenance therapy with fluconazole. Itraconazole (D) is not regarded as first-line treatment of cryptococcal meningitis.

A 19-year-old woman presents to the ED with pain and dyschromatopsia in her right eye. She also describes varying degrees of intermittent paresthesias over the previous month and occasional transient gait disturbance. An MRI shows white matter pathology. Which of the following lumbar puncture findings is associated with her condition? IgM and IgG antibodies to B. burgdorferi Pleocytosis and oligoclonal bands of immunoglobulin G Positive VDRL Xanthochromia

Correct Answer ( B ) Explanation: This patient has multiple sclerosis, which is a neurologic disorder that causes variable motor, sensory, visual, and cerebellar dysfunction as a result of multiple focal areas of CNS demyelination. The patient's orbital pain is likely due to optic neuritis. Dyschromatopsia is the change in color perception and may be more prominent than visual disturbance. Although MRI is the gold standard, lumbar puncture can aid in the diagnosis. The lumbar puncture in approximately 50% of the cases will show pleocytosis, which is an increased number of lymphocytes. In 85%-95% of cases, there will be oligoclonal bands of immunoglobulin G. IgM and IgG antibodies to B. burgdorferi (A) is seen in the CSF in patients with secondary Lyme disease. A positive CSF VDRL (C) is seen in patients with tertiary syphilis. Xanthochromia (D) is a yellow discoloration of the CSF sometimes seen in patients with a subarachnoid hemorrhage.

A 45-year-old man presents to the ED with worsening dysphagia for two days. He complains of difficulty swallowing both solids and liquids. Review of systems is positive for fatigue, mild dyspnea on exertion, and generalized muscle weakness. It is negative for fever, sore throat, cough, or extremity numbness. His vital signs are normal except for a respiratory rate of 32. Physical exam reveals bilateral ptosis, poor inspiratory effort, and proximal extremity weakness. Which of the following tests will confirm his diagnosis? A decrease in bronchial secretions with administration of atropine is a useful diagnostic indicator Administration of edrophonium results in improved respiratory function and resolution of ptosis and muscle weakness Administration of edrophonium results in worsened muscle weakness Elevated antinuclear antibody (ANA) titer Serologic testing that is negative for antibodies to acetylcholine receptors

Correct Answer ( B ) Explanation: This patient has myasthenia gravis (MG), an autoimmune disorder that destroys acetylcholine receptors that leads to poor neurotransmission. Patients report muscle fatigue with a characteristic increase in weakness after repeated muscle use. Weakness improves after rest. Ocular muscle weakness is common leading to ptosis, diplopia, and blurred vision. Dysarthria and dysphagia may also be present. An acute myasthenic crisis is usually triggered by infection, surgery, or rapid cessation of immunosuppressive medications. A patient may present in respiratory distress due to muscle fatigue and require mechanical ventilation. Diagnosis is aided by the edrophonium test. Edrophonium is a short-acting acetylcholinesterase-blocking agent that will increase the availability of acetylcholine at the neuromuscular junction. Patients with a myasthenic exacerbation will show improved signs and symptoms (ptosis resolves). A complication of the edrophonium test is excessive cholinergic symptoms (severe weakness, respiratory distress, increased secretions). When performing an edrophonium test, it is important to have atropine available to help treat any excessive cholinergic symptoms. Atropine (A) has no role in the direct diagnosis of myasthenia gravis. If the administration of edrophoinum (C) worsens symptoms, then it is likely that the patient is having a cholinergic crisis. This occurs in patients being treated for myasthenia gravis who receive too much acetylcholinesterase inhibitor. ANA titers (D) are used in the workup and diagnosis of systemic lupus erythematosis (SLE). Lupus can produce generalized weakness, but muscle strength is not objectively decreased. Serologic testing for antibodies (E) to acetylcholine receptors is useful when positive; however, a negative test does not exclude the disorder.

A 28-year-old woman with myasthenia gravis presents with progressive shortness of breath that started several hours ago. She takes pyridostigmine every 6 hours and has not missed any doses. Yesterday she was prescribed an unknown antibiotic for sinusitis at an urgent care clinic. On exam, her vital signs are within normal limits, and she does not appear in respiratory distress. What is the next immediate step in management? Begin intravenous immunoglobulin (IVIG) Measure negative inspiratory force Obtain a chest X-ray Start corticosteroids

Correct Answer ( B ) Explanation: This patient is experiencing a myasthenic crisis, which is impending respiratory failure with an associated need for mechanical ventilation. Myasthenic crisis may be precipitated by a variety of factors, most often a concurrent infection. It can also follow a surgical intervention, pregnancy, childbirth, or tapering of immunosuppressive medications. In addition, myasthenic crisis can occur spontaneously as part of the natural history of myasthenia gravis (MG) itself. A number of drugs can increase the weakness in myasthenia and should be considered as possible precipitants in this setting. This is of more concern with certain antibiotics (aminoglycosides, erythromycin, and azithromycin); cardiac drugs (beta-blockers, procainamide, and quinidine); and magnesium. An objective measure of respiratory status (i.e., negative inspiratory force [NIF] or forced vital capacity [FVC]) should be trended to evaluate the need for mechanical ventilation. Depending on this patient's respiratory status, she may need to be admitted to the ICU for ventilator management and administration of IVIG and corticosteroids or plasma exchange. Although IVIG (A) and corticosteroids (D) are appropriate treatments for a myasthenic crisis, the next best step is to assess the patient's respiratory status objectively. A chest X-ray (C) is reasonable but should not delay an assessment of respiratory status.

A 17-year-old boy from New Jersey presents with a 1-day history of facial droop. He also complains of a headache. The cutaneous image above represents a rash the patient described having 3 weeks prior to arrival whereas the facial image above represents the current physical examination findings. In addition you note mild nuchal rigidity. A non-contrast head CT is performed and is normal. Which of the following is the next best step for this patient? Acyclovir and corticosteroids Lumbar puncture MRI brain Serologic testing for Lyme disease

Correct Answer ( B ) Explanation: This patient presents with a rash consistent with erythema migrans and facial nerve palsy indicating a likely early disseminated Lyme disease infection and should receive a lumbar puncture to further assess for neurologic disease. Lyme disease is primarily transmitted via the Ixodes scapularis tick. The disease manifests with a number of phases beginning with early Lyme. Early Lyme typically manifests (90% of cases) with the classic erythema migrans rash. The rash begins at the site of the bite and progresses outwards. It typically exhibits central clearing. At this time, patients may also experience non-specific flu-like symptoms. Acute disseminated infection occurs after hematogenous spread of the spirochete and can result in neurologic, cardiac (heart block), arthritis, and ophthalmic manifestations. The most common neurologic manifestation is meningoencephalitis with superimposed cranial neuropathies. CNS Lyme is treated with ceftriaxone and so it is important to obtain a lumbar puncture in patients with neurologic manifestations (eg, headache, nuchal rigidity, vomiting, visual changes, confusion) to rule this in or out. Lumbar puncture will show lymphocytic pleocytosis with a moderately elevated protein level. PCR and cultures can be sent for Borrelia burgdorferi as well. Acyclovir and corticosteroids (A) are often prescribed for Bell's palsy but are not effective in Lyme disease. An MRI of the brain (C) is not necessary as this lesion does not represent a central cranial nerve lesion. Serologic testing for Lyme disease (D) is not recommended in an individual with the classic erythema migrans rash who resides in an endemic area. The patient should be presumed to have Lyme disease, however, it is still important to confirm dissemination into the CNS.

A 20-year-old woman presents with a severe headache that started 10 hours ago. She has never had a headache this intense before. The headache is associated with photophobia and nausea. What management is indicated? Non-contrast head CT and discharge if normal Non-contrast head CT and lumbar puncture Outpatient MRI Symptomatic headache treatment and follow up with neurology if headache resolves

Correct Answer ( B ) Explanation: This patient presents with a severe headache unlike prior headaches raising the suspicion for a subarachnoid hemorrhage (SAH). SAH is defined as the extravasation of blood into the subarachnoid space. Approximately 1-4% of patients presenting to the ED for headache will have a SAH. The classic presentation of SAH is a patient presenting with a "thunderclap" headache or "the worst headache of life." However, it should be suspected in any patient who presents with an acute, severe headache that reaches maximal intensity in a short period of time or is different than prior headaches. Although some patients will present with traumatic SAH, the most dangerous form is SAH related to berry aneurysms. These aneurysms are prone to catastrophic rupture. Many patients who have large berry aneurysm bleeds will report a sentinel headache, which may or may not have prompted a doctor visit. It is vital to diagnose patients presenting with a sentinel bleed as neurosurgical intervention for berry aneurysms can avoid catastrophic events. Non-contrast CT scan of the head should be ordered in any patient with a presentation concerning for SAH. The sensitivity of CT within the first 6 hours of headache onset is extremely high (> 95%) but the sensitivity decreases with time. Overall sensitivity is about 93%. A negative CT scan should be followed by a lumbar puncture (LP). The LP is performed looking for the presence of blood in the CSF or xanthochromia (the yellowish pigmentation of CSF secondary to hemoglobin metabolism). Xanthochromia takes approximately 12 hours to develop. Non-contrast head CT scan (A) alone is highly sensitive for the diagnosis of SAH if performed within the first 6 hours. However, because SAH is a life-threatening disease if missed, current guidelines recommend against using non-contrast head CT to rule out the diagnosis. An MRI of the brain (C) can be helpful in detecting berry aneurysms as the cause of SAH but the workup should not be delayed when SAH is suspected. Symptomatic treatment (D) should be given to all headache patients regardless of etiology. However, response to treatment does not rule out dangerous pathology.

A 20-year-old woman presents with numbness in the left arm, left eye pain and blurry vision, and generalized weakness. She states she has had similar episodes in the past. Physical examination reveals a visual acuity of 20/20 in the right eye and 20/100 in the left eye. She also has decreased sensation in the left arm. Which of the following is true about the patient's disease? It is more common in men than women Lumbar puncture will reveal CSF pleocytosis and elevated IgG Lumbar puncture will reveal increased opening pressure Non-contrast head CT is diagnostic

Correct Answer ( B ) Explanation: This patient presents with multiple neurologic symptoms and findings that cannot be accounted for by a single lesion, which is highly suggestive of multiple sclerosis (MS). MS is an inflammatory disease of the central nervous system (CNS). The inflammatory disease involves demyelination of regions within the CNS with sparing of axons. The clinical picture is highly variable and typically involves symptoms occurring in different parts of the body, which are intermittent, recurrent and "separated by time and space." Common symptoms include unilateral vision changes or loss, optic neuritis, sensory loss, weakness, paresthesias, ataxia, bladder dysfunction, diplopia, and depression. Intranuclear ophthalmoplegia accounts for diplopia. Lhermitte sign, an electrical shock type sensation radiating down the back of the neck with flexion, is common as well. A common complaint is worsening of current symptoms with a change in temperature (i.e. fever, exercise or a warm shower). This is referred to as Uhthoff's phenomenon. Analysis of CSF is abnormal in 90% of cases with pleocytosis (50%) and elevated gamma globulin (70%) being the most common findings. MRI of the brain and spinal cord is a sensitive test that will show plaques (demyelination) in 95% of patients.

A 21-year-old woman presents with progressive headaches for one month. She also complains of intermittent blurred vision bilaterally and increased headache with coughing. Direct ophthalmoscopy findings are shown above. What abnormality is typically found on lumbar puncture in patients with this presentation? Decreased glucose Elevated opening pressure Elevated red blood cell count Elevated white blood cell count

Correct Answer ( B ) Explanation: This patient presents with signs and symptoms consistent with a diagnosis of idiopathic intracranial hypertension (IIH), which can be diagnosed by an increased opening pressure on lumbar puncture. IIH, previously called pseudotumor cerebri and benign intracranial hypertension is a relatively common disease. It is typically seen in obese women. The exact pathophysiology of the disease is not fully understood but it seems to result from a decreased outflow of cerebrospinal fluid (CSF) and increased brain water content. The headache is typically gradual in onset and generalized. It may be worsened by eye movement or valsalva maneuver (or other maneuvers that increase intracranial pressure). Visual complaints are common and can range from transient blurring of vision to vision loss. The headache may also be accompanied by nausea, vomiting and dizziness. Physical examination reveals papilledema and patients may also have visual field defects. Diagnosis is made by an increased opening pressure on lumbar puncture. Opening pressure will be elevated (above 250 mm H2O). Decreased glucose (A) and elevated white blood cell count (D) are typically seen in meningitis. An elevated red blood cell count (C) is found in subarachnoid hemorrhage

A 14-year-old boy presents with headache, fever and altered mental status. He was recently seen and evaluated for sinusitis one week ago, but was not given antibiotic treatment at that time. A CT scan of the head is performed. What management is indicated? Ceftriaxone, metronidazole and admit Ceftriaxone, metronidazole and neurosurgery consultation Ceftriaxone, vancomycin and neurosurgery consultation Pyramethamine, sulfadiazine and admit

Correct Answer ( B ) Explanation: This patient presents with signs, symptoms and imaging consistent with an intracranial abscess. Immediate management involves administration of antibiotics covering the most likely pathogens and neurosurgical consultation. CNS abscess is unusual in immunocompetent hosts but can result from direct spread from other infections. The most common causes from direct spread are dental infections, otitis media and sinus infections. Patients with multiple abscesses should raise suspicion for endocarditis. Streptococcus species and anaerobic bacteria are the predominant causative agents. Patients with a history of trauma or intracranial surgery are also at risk for MRSA. CT scan is the diagnostic modality of choice and typically is followed by a lumbar puncture (unless there are signs of increased intracranial pressure), which aids in determining the etiologic agent. Location, size and number of abscesses dictate management. Patients with a single abscess are more readily treated with neurosurgical intervention while those with multiple abscesses are usually treated with antibiotics alone. Initial antibiotics in patients without a history of neurosurgery should be with a third generation cephalosporin and an agent covering anaerobic bacteria like metronidazole. Patients with CNS abscesses should not be admitted (A) without neurosurgical consultation as many will require emergent surgery. Ceftriaxone and vancomycin (C) is inadequate coverage of the typical bacterial agents. Pyramethamine and sulfadiazine (D) are typically used in the treatment of CNS toxoplasmosis, which appears as ring enhancing lesions on CT scan.

An 18-year-old man presents with lower extremity weakness for 4 days. He states that he had a bout of diarrhea 2 weeks ago that resolved but then began feeling weak in his legs. Over the last 24 hours, he has begun to feel weak in his arms and has difficulty breathing. Physical examination reveals 3/5 strength in both lower extremities and 4/5 strength in both upper extremities along with decreased deep tendon reflexes. What management is indicated for the suspected diagnosis? Ceftriaxone and vancomycin Intravenous immunoglobulin MRI of the brain Oral steroids

Correct Answer ( B ) Explanation: This patient presents with symptoms and signs concerning for Guillain-Barre syndrome (GBS) and should be treated with intravenous immunoglobulin (IVIG). GBS is a peripheral nerve disorder with an unpredictable and heterogeneous clinical course. The most common form is the acute inflammatory demyelinating polyneuropathy (AIDP) accounting for 90% of cases. Most patients present with symptoms days to weeks after the resolution of a gastrointestinal or upper respiratory illness. Symptoms are worse in the lower extremities than in the upper extremities and typically, weakness in ascending. The descending version is called the Miller Fisher variant. In addition to the progressive, symmetric weakness, patients will have variable sensory findings and decrease in deep tendon reflexes. Notably, anal sphincter tone is spared. Respiratory compromise is common and approximately 33% of patients will require intubation and mechanical ventilation. The mortality and recurrence rate are both 3%. All patients presenting with symptoms concerning for GBS should have vital capacity and negative inspiratory pressure (NIF) measured frequently as a decrease predicts the need for intubation. Lumbar puncture typically shows elevated protein with a mild pleocytosis. Treatment should be started with either IVIG or plasma exchange. GBS is not the result of an ongoing infection so treating with antibiotics (A) will not help. MRI of the brain (C) does not add to diagnosis or management as this is a disease of peripheral nerves. Oral steroids (D) have been shown to delay recovery

An 8-year-old previously healthy boy presents with back pain and fever for 4 days. He complains of pain in the low back, which is increased with bending or twisting. The pain radiates down into his left leg. He denies trauma. Physical exam is remarkable only for tenderness to palpation over the lumbar spine. What management is indicated? Ibuprofen and follow up with his pediatrician MRI of the lumbar spine Plain radiographs of the lumbar spine Urinalysis

Correct Answer ( B ) Explanation: This patient presents with symptoms consistent with discitis and should have an MRI for diagnosis. Discitis is a rare infection of the nucleus pulposus and can involve the cartilaginous endplate and vertebral body. It can occur spontaneously or after surgical procedures. Typically, patients present with severe pain localized to the spinal level of involvement. Any movement of the spine exacerbates the pain. Children often present with sudden onset of back pain and refusal to walk. The lumbar spine is most commonly involved and the average age of patients is 7 years. Radicular symptoms are common (present in 50-90% of cases); however, fever rates have ranged in multiple studies with percentages from 50-90% of patients being febrile. Neurologic deficits are uncommon. Serious pathology should be suspected in any pediatric patient presenting with back pain. Neurologic symptoms are rarely present. MRI is the best study for diagnosis and can also rule out other critical diagnoses including epidural abscess. Laboratory studies are non-specific and insensitive but typically will have an elevated erythrocyte sedimentation rate. White blood cell counts are frequently within the normal range. Symptomatic treatment with ibuprofen and follow up (A) is inadequate management because the higher likelihood of serious pathology in pediatric patients presenting with back pain in comparison to adults. Plain radiographs of the spine (C) are unlikely to be helpful early in the clinical course but may reveal disk space destruction after 2 to 4 weeks of symptoms. Urinalysis (D) is not indicated as the patient does not have symptoms consistent with a urinary tract infection

A 67-year-old woman presents with severe unilateral pain in her face and head. She characterizes the symptoms as lasting only a few seconds and denies any associated nausea, vomiting, dizziness, or blurry vision. Her Head, Eye, Ear, Nose and Throat Exam (HEENT) and neurologic exam are normal. Which of the following statements is correct regarding this patient's condition? Facial droop is associated with this condition Pain is most commonly located in the V2 and V3 cranial nerve distribution Pain is usually bilateral in the elderly population Poor dentition is associated with this condition The typical pain attack lasts 30-60 minutes then resolves spontaneously

Correct Answer ( B ) Explanation: Trigeminal neuralgia manifests with unilateral facial pain characterized as lancinating paroxysms of pain in the lips, teeth, gums, and chin. The pain is often associated with triggers such as chewing, brushing the teeth, shaving, washing or touching the affected area of the face, swallowing, or exposure to heat or cold in the affected area. Pain most often occurs in the maxillary (V2) and mandibular (V3) distribution of the trigeminal (V) nerve. The ophthalmic division (V1) is rarely involved. Facial droop (A) never accompanies trigeminal neuralgia. Other than acute stroke, conditions associated with facial droop are Bell's palsy (painless) and Ramsey-Hunt syndrome (painful). The pain (C) of trigeminal neuralgia is usually unilateral, a feature independent of age. Poor dentition (D) is not associated with trigeminal neuralgia; however, odontogenic infections may cause unilateral facial pain and should be ruled out in such patients. Pain occurs in clustered episodes (E) that last a few seconds to several minutes. The attacks can occur during the day or night but rarely during sleep.Facial droop (A) never accompanies trigeminal neuralgia. Other than acute stroke, conditions associated with facial droop are Bell's palsy (painless) and Ramsey-Hunt syndrome (painful). The pain (C) of trigeminal neuralgia is usually unilateral, a feature independent of age. Poor dentition (D) is not associated with trigeminal neuralgia; however, odontogenic infections may cause unilateral facial pain and should be ruled out in such patients. Pain occurs in clustered episodes (E) that last a few seconds to several minutes. The attacks can occur during the day or night but rarely during sleep.

A 65-year-old man presents to the ED complaining of persistent dizziness. He complains of nausea and feels like the room is spinning. He has a history of hypertension. Vital signs are within normal limits. Which of the following physical exam findings supports a central cause of his dizziness? Hearing loss Limb ataxia Patient remains steady when eyes are open and closed when performing the Romberg test Spontaneous nystagmus that is suppressed by visual fixation Sudden onset of intense, intermittent episodes of disequilibrium

Correct Answer ( B ) Explanation: Vertigo is the perception or sensation of movement. This may be described by the patient as a feeling of swaying, spinning, whirling, leaning, or tilting. The patient may also report feeling intoxicated. Nausea and vomiting may also be seen. Vertigo results from dysfunction in the vestibular system from either its peripheral or central components. Physical examination will reveal nystagmus in nearly all patients, often lateralizing to the affected side. Although less common than peripheral etiologies, central causes of vertigo are more concerning and include conditions such as vertebrobasilar insufficiency, brainstem and cerebellar infarct or hemorrhage, basilar artery migraine, and degenerative diseases such as multiple sclerosis. In general, when compared to peripheral vertigo, the symptoms of central vertigo are less acute and more persistent and may be associated with neurologic deficits. However, exceptions exist, particularly in those patients with cardiovascular risk factors. Limb ataxia is not seen with peripheral causes and is usually attributable to cerebellar lesions. A noncontrast head CT is a reasonable screening test with suspected central vertigo (and is most appropriate to evaluate for potential intracranial hemorrhage). Secondary to the fact that it does not permit adequate visualization of the cerebellum, it is not considered the definitive test. Magnetic resonance imaging with angiography is the study of choice in most cases of suspected central causes of vertigo.

A 24-year-old man who emigrated from Mexico five years ago presents to the ED via EMS after he had a seizure at home. His witness noticed tonic-clonic activity that lasted about one minute. The patient has no past medical history. Vital signs are BP 138/89, HR 105, T 37.7F, RR 18, and pulse oximetry 100% on room air. He is somnolent but responds to verbal command and is moving all four extremities purposefully. A CT scan is obtained and seen above. Which of the following is the most likely diagnosis? Herpes encephalitis Meningioma Neurocysticercosis Toxoplasmosis

Correct Answer ( C ) Explanation: Cysticercosis is caused by the larval form of the tapeworm Taenia Solium, a common CNS pathogen found in many tropical areas. It is acquired by humans who swallow eggs found in the feces of a person who has intestinal tapeworm. The adult worm matures in the small intestine. The larval form may penetrate through the gut wall and end up any where in the body. The most common sites include the CNS, muscle, and soft tissue. In the brain, the larvae form an expanding cyst that induces an intense immunologic reaction from the host. Neurologic symptoms begin when the involved neural tissue cannot accommodate the enlarging cyst. Seizure activity often is the first indication of cysticercosis, which should be considered in any adult patient with undiagnosed seizures, especially immigrants. CT scan with contrast or MRI may reveal a ring-enhancing lesion. These lesions may mimic a CNS abscess, metastasis, or primary tumor. Benzodiazepines are used to treat an actively seizing patient. Albendazole is the treatment of choice for Taenia Solium. Corticosteroids may be necessary if there is edema due to CNS cysts. Neurologic consultation is recommended for neurocysticercosis because obstructive hydrocephalus is a complication. Meningioma (B) is a benign tumor that is usually asymptomatic when less than 2.0 cm in size. It is typically an incidental finding on neuroimaging that is located near the inner surface of skull or tentorial dura. Herpes encephalitis (A) can present with fever and seizures, however a ring-enhancing lesion is not present on imaging. HSV encephalitis is associated with CT scan changes in the temporal and frontal lobes. Toxoplasmosis (D) is a protozoan infection by Toxoplasmosis gondii. This typically leads to focal encephalitis in AIDS patients most commonly with a CD4 count below 100 cells/mL. Contrast enhancing CT typically shows ring -enhancing lesions with toxoplasmosis, too.

A patient is diagnosed with Campylobacter jejuni enteritis, receives treatment and recovers. Two weeks later, he presents with weakness in his legs followed by weakness in his arms. He denies dyspnea, but reports diplopia and dysphagia. Examination reveals a mild fever, extraoccular muscle palsies, hyporeflexia, distal paresthesias and intact proprioception. Laboratory testing is significant for a normal CBC and a PaO2 of 88 mmHg. Which of the following is the most appropriate intervention at this time? Continuous positive airway pressure (CPAP) Corticosteroids Plasmapheresis Vitamin B12

Correct Answer ( C ) Explanation: Guillain-Barre syndrome (GBS) is a symmetric, progressive ascending muscle weakness that usually starts in the legs and may be acute or subacute. The condition is life-threatening if respiratory or swallowing muscles are involved. GBS can follow minor respiratory or GI illness, inoculation, or surgical procedures. It carries a poorer prognosis when it follows Campylobacter jejuni infection. The clinical hallmark of GBS is lack of deep tendon reflexes. Patients experience weakness of >2 limbs that typically begins with the proximal lower extremities. As weakness progresses, patients may experience shortness of breath, constipation, facial weakness, dysphagia, ophthalmoplegia, dysarthria. And sensory disturbances. Lumbar puncture results will demonstrate increased CSF protein but a normal cell count. Treatment is supportive. Plasmapheresis or IVIG may improve recovery time and decrease residual neurologic effects. CPAP (A), or other ventilatory assist device, is not recommended in a patient without dyspnea or with a normal arterial oxygen partial pressure. CPAP is more commonly used for obstructive sleep apnea than acute ventilatory support. Several controlled studies have shown corticosteroids (B) to be of no benefit. Vitamin B12 (D) deficiency causes a subacute or chronic syndrome of neurologic symptoms. It mainly affects the dorsal and lateral spinal cord columns leading to distal paresthesias, imbalance and altered vibrioception and proprioception. It occurs with macrocytic megaloblastic (pernicious) anemia. Chronic cases are associated with dementia. Folate deficiency can also present in a similar fashion with similar spinal cord pathology.

Answer: 5-10 minutes; it is a short-acting acetylcholinesterase-blocking agent.

Correct Answer ( C ) Explanation: Guillain-Barré syndrome (GBS) is a polyneuropathy characterized by symmetric weakness typically beginning distally and spreading proximally. Sensory abnormalities may also occur although motor weakness predominates. Some patients present days to weeks after an antecedent upper respiratory or gastrointestinal illness. GBS is diagnosed clinically with the assistance of some additional tests including a lumbar puncture. The results of the lumbar puncture demonstrate a markedly elevated protein and mild pleocytosis. A CT myelogram (A) is performed in place of MRI for patients who cannot have an MRI. This would be considered in the evaluation of cord compression or tumors. It is not helpful in the evaluation of GBS. New onset myasthenia gravis is diagnosed by both the clinical symptoms of muscle weakness and fatigability in combination with a positive edrophonium test (B). An antibody against the nicotinic acetylcholine receptors at the neuromuscular junction causes myasthenia gravis. Edrophonium is an acetylcholinesterase inhibitor leading to higher levels of acetylcholine at the neuromuscular junction. When administered, weakness (particularly ptosis) improves. Pulmonary function tests (D) are helpful in the prognostication of patients with GBS. As the weakness progresses more proximally, the muscles of respiration are affected and patients may require mechanical ventilation.

A 67-year-old man with a history of hypertension and diabetes mellitus presents to the emergency department with sudden onset of left leg weakness. His examination is notable for 3/5 strength in his left lower extremity as well as diminished sensation to temperature and light touch. Which of the following vascular territories is most likely affected in this patient? Left lenticulostriate arteries Left posterior cerebral artery Right anterior cerebral artery Right middle cerebral artery

Correct Answer ( C ) Explanation: Ischemic strokes are the most common type of cerebrovascular accident and can be caused by thrombosis, embolism, and hypoperfusion. Thrombosis is the most common cause of ischemic stroke. Risk factors for ischemic stroke include smoking, increasing age, male sex, diabetes, hypertension, and hyperlipidemia. The middle cerebral artery is the artery most commonly affected by ischemic strokes. Anterior cerebral artery (ACA) infarcts are more rare due to collateral blood flow supplied by the anterior communicating artery. The ACA supplies the frontal, prefrontal, and supplementary motor cortex. Symptoms of an ACA infarction include contralateral lower extremity weakness and sensory loss with sparing of the upper extremity and face. Management of an ACA stroke depends on the timing of presentation, with administration of tPA improving outcomes in eligible candidates. Management of all stroke syndromes should include reduction of risk factors, including optimization of blood sugar, smoking cessation, and control of blood pressure and cholesterol. The lenticulostriate arteries (A) are small penetrating arteries that supply the deep structures of the brain. Ischemia of the left lenticulostriate arteries would cause right-sided symptoms, which can be motor, sensory, or both depending on the deep brain structure affected by the stroke. The posterior cerebral artery (B) supplies the occipital lobe. Symptoms of a posterior cerebral artery stroke are subtle and may include ataxia, vertigo, contralateral motor weakness, and visual field cuts. The middle cerebral artery (D) is the artery most commonly involved in ischemic stroke. Symptoms depend on what area of the artery is involved as well as which lobe is dominant in the patient. Typical symptoms include contralateral hemiparesis (involving both the arm and leg, in contrast to an ACA stroke), facial droop, and sensory loss. Involvement of the dominant lobe will result in aphasia, which may be expressive, receptive, or both.

What is the most common cause of an intracranial neoplasm? Astrocytoma Meningioma Metastases Pituitary adenoma

Correct Answer ( C ) Explanation: Metastatic disease (primarily from lung cancer) is the most common cause of an intracranial neoplasm. Other cancers that commonly metastasize to the brain include breast and colon carcinoma. The malignant gliomas (A) (anaplastic astrocytomas and glioblastoma multiforme) are the most common glial tumors. These are located in the cerebral hemispheres. Meningiomas (B) are generally benign, slow-growing tumors that originate in the meninges. Pituitary adenomas (D) are tumors originating from the pituitary gland and are often first noted when the patient exhibits visual impairment from compression of the optic chiasm.

Which of the following is the most common type of migraine headache? Basilar-type migraine Migraine with aura Migraine without aura Ophthalmoplegic migraine

Correct Answer ( C ) Explanation: Migraine headaches tend to be familial and affect women twice as often as men. The underlying pathophysiology is thought to be vasogenic inflammation. The first headache usually occurs in an individual in the teens or twenties. The headache is often unilateral, pulsating in quality, moderate to severe in intensity, and exacerbated by routine activities. The side of the headache can vary with individual attacks and may be bilateral up to 40% of the time. The onset is usually gradual and the attacks last from 4-72 hours. The migraine without aura is most common (80%) and is usually with associated nausea, vomiting, photophobia, or phonophobia. Pharmacological treatment is divided into abortive therapies (limit the intensity and duration of an episode) and prophylactic therapies (decrease the frequency and intensity of attacks). Basilar-type migraines (A) are associated with an aura caused by brainstem involvement and are associated with multiple neurologic findings (visual impairment, dysarthria, tinnitus, vertigo, bilateral paresthesias, paresis, and altered mental status). The aura of a classic migraine (B) consists of focal neurologic symptoms that precede and portend the migraine attack. The aura is reversible and lasts 10-20 minutes but may continue up to an hour. The most common aura is visual involving scintillating scotomas (bright rim around an area of visual loss), teichopsias (subjective visual image perceived with eyes open or closed), fortification spectrums (zigzagged wall of fortress slowly drifting across visual field), photopsias (poorly formed brief flashes or sparks of light), and blurred vision. Ophthalmoplegic migraine (D) is a rare syndrome associated with paresis of 1 or more ocular nerves—most commonly CN III (oculomotor). Secondary causes should be evaluated for, including intracranial aneurysm and mass lesion.

Which of the following is characteristic of myasthenia gravis? Descending paralysis Improvement of muscle strength with repeated stimulation Positive ice bag test Ptosis worse in the morning

Correct Answer ( C ) Explanation: Myasthenia gravis is caused by the autoimmune destruction of the nicotinic acetylcholine receptors on the postsynaptic membrane. As a result of this autoantibody destruction, over time less total receptors are available. At the remaining receptors, the autoantibodies are competing with acetylcholine for the binding sites. With repeated stimulation, fewer receptor sites are both physically available because of destruction and competition, but also because many remain in the refractory period. As a result, muscles weaken and fatigue, which is the hallmark of the disease. Most commonly the first manifestation of the disease is ocular symptoms including ptosis, diplopia and blurred vision. Over the course of the disease, involvement of the respiratory and bulbar muscles may develop. In order to make the diagnosis, 2 options exist. The first is a bedside edrophonium test although the production of this compound is discontinued. Edrophonium is an acetylcholinesterase blocker and therefore causes an accumulation of acetylcholine in the synapse improving symptoms. Symptoms of myasthenia gravis improve with cooling and so the ice bag test is another alternative diagnostic bed-side maneuver. In both scenarios, the distance between the upper and lower eyelids with ptosis is measured. The ice bag test is based on the physiologic principle of improving neuromuscular transmission at lower muscle temperatures, the eyelid muscles are the most easily cooled by the application of ice. After the application of either the ice bag or administration of edrophonium, the distance between the lids is re-measured to assess for a difference. Descending paralysis (A) is classic for botulism in which the botulinum toxin irreversibly binds to presynaptic membranes preventing the release of acetylcholine into the synapse. Botulism occurs in three forms to cause disease in humans: food-borne, wound botulism and infant botulism. Improvement of muscle strength with repeated stimulation (B) is characteristic of Lambert-Eaton syndrome (LES). In LES, antibodies target the calcium channels which results in a reduction of acetylcholine release. With repeated stimulation, more acetylcholine is released from the intact portions of the synapse causing improvement of muscle strength. In MG, ptosis worsens over the course of the day, not in the morning (D).

An 18-year-old college student presents from his dormitory to the emergency department with headache, fever, and stiff neck for two days. He thought he had a cold and has been taking acetaminophen without relief of his headache. He has no medical history. His parents did not have him vaccinated because of worries about autism. Vital signs are T 38.8.°C, BP 110/50 mm Hg, HR 137/min, and RR 25/min. He is sleepy but arousable. When you flex the patient's head, you note flexion of the patient's lower extremities (hips and knees). Skin exam reveals diffuse petechiae. What is the most likely diagnosis and recommended therapy? Cryptococcus; amphotericin B and flucytosine Herpes simplex virus; acyclovir Neisseria meningitidis; ceftriaxone plus vancomycin Streptococcus pneumoniae; ceftriaxone plus vancomycin West Nile virus; supportive care

Correct Answer ( C ) Explanation: Neisseria meningitidis is a common cause of meningitis in crowded living conditions such as college dorms and military barracks. The classic signs of meningitis are fever, headache, neck stiffness, and mental status change. Petechiae and purpura may also be seen with more advanced cases of meningococcemia. N. meningitidis is an encapsulated bacterium that gains entry through the upper airway and spreads hematogenously to the subarachnoid space. Elements of the capsule trigger an inflammatory cascade, causing the brain and meninges to become edematous, leading to decreased CSF drainage. Disruption of the blood-brain barrier also allows entry of protein and water into the brain, resulting in vasogenic edema. The Brudzinski sign is positive if flexion of the neck results in passive flexion of the hips and knees. It is more sensitive than the Kernig sign. Diagnosis of meningitis is made with lumbar puncture (LP). The LP typically reveals an opening pressure > 30, WBC > 1000 with > 80% PMNs, glucose < 40 mg/dL, protein > 200 mg/dL, and cultures test positive for gram negative bacteria. Early treatment of meningitis is essential. Antibiotics should be administered as quickly as possible, even before lumbar puncture. Cryptococcal meningitis (A) is an infection of immunocompromised patients. It is considered an AIDS-defining illness but does not occur exclusively in patients with HIV. It may present with fever, headache, mental status changes, and focal neurologic deficits. Meningismus is uncommon. If cryptococcal infection is suspected, be sure to check an opening pressure on LP. Cryptococcus in the CSF is diagnosed by positive cryptococcal antigen, positive culture, or positive staining with india ink. Treatment is with amphotericin B and flucytosine. Herpes simplex virus (B) is an uncommon cause of viral meningitis in patients with an intact immune system. Most viral meningitides are caused by enteroviruses (echovirus, coxsackievirus, enterovirus). Streptococcus pneumoniae (D) is the most common cause of bacterial meningitis in adults. It is an encapsulated organism that enters the body via the upper respiratory tract. It has many of the same features as N. meningitis, but it is not associated with dormitory living and does not result in the purpura of fulminant meningococcemia. Treatment is with ceftriaxone and vancomycin. West Nile virus (E) causes encephalitis. It is an arbovirus, transmitted by mosquito bite. The virus replicates outside the brain and crosses the blood-brain barrier during viremia. It primarily affects the gray matter, resulting in cognitive dysfunction, psychiatric abnormalities, lethargy, and seizures. Headache and fever are usually present. Treatment is mainly supportive.

An 80-year-old homeless man is found in his riverside tent with confusion and fever in July. On physical examination, the patient is disoriented with T 38.2°C, HR 122 bpm, BP 106/84 mm Hg, and oxygen saturation 99% on room air. You note splenomegaly, flaccid paralysis, and multiple mosquito bites. Which lab abnormality would lead you to suspect West Nile encephalitis in this patient? Decreased CSF glucose level Leukocytosis Lymphopenia Markedly increased serum ALT/AST

Correct Answer ( C ) Explanation: Patients with West Nile encephalitis (WNE) will have leukopenia with a pronounced and prolonged lymphopenia, which can aid in distinguishing it from other causes of encephalitis. WNE is an arthropodborne virus (arbovirus) endemic to the Middle East and now found throughout the United States. It is transmitted by the Aedes mosquito, with birds serving as the intermediate host. Most people with WNE will remain asymptomatic or have a mild viral syndrome. Patients with encephalitis will present with new psychiatric symptoms, cognitive deficits, seizures, flaccid paralysis, and tremors. Most will have a headache and low-grade fever. In patients with meningoencephalitis, a lumbar puncture will show a pleocytosis with mostly lymphocytes, normal to elevated glucose, and increased protein. Decreased CSF glucose (A) is found in patients with bacterial or fungal meningitis. It is generally normal in viral meningitides. A mildly increased ALT/AST (D) may be seen in WNE, Epstein-Barr virus, Rocky Mountain spotted fever, and ehrlichiosis; however, markedly elevated transaminases are not generally part of arboviral infection. Leukocytosis (B) in a patient with WNE suggests an overlying bacterial infection.

A 24-year-old woman presents to the emergency department with a headache and receives a lumbar puncture. She is eventually discharged home in improved condition, but returns 24 hours later with a worsened headache, now throbbing in nature, which is worse in the standing position and relieved in the supine position. Which of the following reduces the risk of post-lumbar puncture headache? Inserting the needle bevel perpendicular to the spine Lying supine for one hour after the lumbar puncture is completed Using a higher gauge needle Using a Quincke needle

Correct Answer ( C ) Explanation: Post-lumbar puncture headache is the most common complication of a lumbar puncture (LP). The headache usually presents within the first 48 hours after LP and tends to be worsened by standing up and improved by lying down. The exact etiology is unclear, but may be related to the persistent leak of cerebrospinal fluid from the dural puncture site. Post-LP headache is generally benign and self-limiting within 24 to 48 hours, but can in some cases be debilitating. Bed rest and intravenous caffeine have been shown to improve symptoms, as has epidural blood patching, though the invasiveness of this procedure leads most providers to reserve it only for the most intractable and debilitating cases. Prevention of post-LP headache has been the subject of much research, with mixed and sometimes inconsistent results. Modern evidence-based strategies for prevention include the use of a higher gauge spinal needle (which corresponds to a smaller caliber needle), orientation of the needle's bevel parallel to the spine, use of a noncutting needle like the pencil-point Whitacre or Sprotte models, and reinsertion of the stylet prior to needle removal. The Quincke needle (D) is another name for the standard cutting spinal needle and is not associated with decreasing the risk of post-LP headache. The noncutting Whitacre and Sprotte needles are postulated to separate the dural fibers rather than cutting them. Orientation of the needle bevel parallel to the spine, not perpendicular to the spine (A), has been shown to lower post-LP headache incidence, possibly because it cuts the dural fibers in a less traumatic manner. Recent meta-analyses have not shown lying supine for one hour after LP (B) reduces the risk of post-LP headache.

A patient presents with dizziness. Romberg testing with the eyes open is normal, but with the eyes closed, the patient sways and almost falls over. Given this Romberg test result, pathology in which of the following systems is most likely? Cardiovascular Cerebellar Vestibular Visual

Correct Answer ( C ) Explanation: Romberg testing analyzes the balance system, which is comprised of incoming vestibular input from the inner ear, visual input from the eyes and proprioceptive input form the joints and muscles. The brain and cerebellum integrate these signals and compensate with outgoing motor commands. In essence, balance requires a harmony between 5 systems: vestibular, visual, peripheral nervous, central nervous and musculoskeletal. During a Romberg test, the patient stands with feet together and eyes open and hands on hips or shoulder abduction. The clinician then monitors for 30 seconds, comparing the patient to something perpendicular behind the patient. A sway or falling-over fosters a "positive with eyes open at X seconds" result. If necessary, the patient is then corrected, and the test begins again, but this time with the patient's eyes closed. A sway or falling-over here fosters a "positive with eyes closed at X seconds" result. If a Romberg test is positive with eyes open, significant pathology exists, and could include insults to any of the 5 systems above. Classically, a positive Romberg test with the eyes closed points towards a vestibule-labyrinth (inner ear), proprioception (dorsal columns) such as tabes dorsalis, or peripheral nerve sensory abnormality such as chronic inflammatory demyelinating polyradiculopathy (chronic form of Guillain-Barre' syndrome).

You are caring for a 60-kg patient who has been seizing for 30 minutes. You have already administered 4 mg of IV lorazepam and 1200 mg of phenytoin without termination of seizure activity. Which of the following should most likely be your next step? Administer another bolus of IV phenytoin Administer IV fosphenytoin Administer IV pentobarbital Administer IV sodium bicarbonate

Correct Answer ( C ) Explanation: Seizing patients become apneic and may vomit, compromising their airway and putting them at significant risk for hypoxia and aspiration. Failure to respond to optimal benzodiazepine and phenytoin loading functionally defines refractory status epilepticus. At this point, a third line agent such as pentobarbital (15 mg/kg IV) should be administered. Use caution when adding barbiturates to benzodiazepines because their co-administration may potentiate respiratory depression. Propofol is an alternative to pentobarbital and has the advantages of a short half-life and rapid clearance. These agents are given by IV drip and titrated to a burst-suppression pattern in the EEG tracing. Consider also other causes of benzodiazepine-resistant seizures such as isoniazid toxicity (treated with pyridoxine) and hyponatremia. Phenytoin (A) dosing is 20-30 mg/kg. This patient has already received a 20 mg/kg bolus. Additionally, phenytoin can lead to toxicity from its diluent propylene glycol. It is best to use another agent. Fosphenytoin (B) is a prodrug of phenytoin that can be infused at a faster rate. However, it is prudent not to administer in the setting of full-dose phenytoin due to the risk of toxicity. Although most patients in refractory status epilepticus will have a metabolic acidosis, sodium bicarbonate (D) does not play a role in treating seizures

A 20-year-old man is seen in the emergency department for sudden onset left-sided severe headache with associated syncope, nausea, and vomiting. He has a history of migraines but states this feels different from his usual headaches. A non-contrast CT scan of his head today is negative for hemorrhage. What is the most appropriate next step in management? AEmergent MRI BNeurosurgery consult for aneurysm clipping CPerform a lumbar puncture DRepeat a CT scan with contrast

Correct Answer ( C ) Explanation: Subarachnoid hemorrhage (SAH) classically presents with a sudden severe "thunderclap" headache which patients may describe as "the worst headache of my life." The headache can be lateralized and may be associated with a syncopal episode, nausea, and vomiting. Vitreous (pre-retinal) hemorrhage, called Terson's syndrome, or seizure may also be present and are poor prognostic indicators. Almost half of patients report a sudden and severe headache one to three weeks before the SAH event. These "sentinel headaches" are thought to represent a minor bleed. Change from a patient's typical headache or a thunderclap headache should prompt immediate and emergent evaluation by non-contrast head CT. Non-contrast CT has high sensitivity for SAH (almost 100 percent); however, CT may miss small bleeds or those with atypical symptoms, such as isolated neck pain. Considering the disastrous and likely fatal consequences of missing this diagnosis, lumbar puncture is recommended in patients with a negative initial CT in whom SAH is a concern. Studies have shown lumbar puncture to be 100 percent sensitive in the detection of SAH missed by CT scan. Emergent MRI (A) does not have increased sensitivity in the detection of SAH when compared to CT. The use of contrast with a CT scan (D) obscures the presence of blood in the subarachnoid space, making it almost impossible to diagnose a hemorrhage. While most subarachnoid hemorrhages are caused by ruptured saccular aneurysms, emergent aneurysm clipping (B) would be inappropriate without a diagnosis of SAH or knowledge of the location of the aneurysm.

63-year-old man is brought to the ED by paramedics secondary to aphasia that began 30 minutes prior to arrival. The man is unable to give any history and has never been to your hospital. His vital signs are T 37°C, BP 180/98, HR 90, and RR 20. His exam is unremarkable except for profound expressive aphasia. Capillary blood glucose is 124 mg/dL. His ECG shows normal sinus rhythm with normal ST segments and T waves. CT scan of the head is normal. Sixty minutes after arriving to the ED, the patient's aphasia resolves. Which of the following are the most appropriate diagnosis, treatment, and disposition for this patient? Hypertensive emergency; start IV antihypertensive therapy and admit to the medical intensive care unit Stroke; administer tissue plasminogen activator; admit to stroke unit Transient ischemic attack; administer aspirin and dipyridamole; admit to stroke unit Transient ischemic attack; administer low molecular weight heparin and warfarin; discharge home with close neurology follow up Transient ischemic attack; start heparin infusion; admit to stroke unit

Correct Answer ( C ) Explanation: The 2009 AHA/ASA guidelines define transient ischemic attack (TIA) as "a transient episode of neurological dysfunction caused by focal brain, spinal cord, or retinal ischemia without acute infarction." Previously, TIAs were differentiated from strokes by the duration of symptoms—with TIAs lasting less than 24 hours. However, it has since become evident from imaging studies that many patients with symptoms lasting less than 24 hours have had a stroke. The latest class IA treatment recommendation is for aspirin alone or aspirin plus dipyridamole. Patients diagnosed with a TIA should be admitted for assessment of stroke risk factors because the 90-day risk of subsequent stroke is ~ 9.5%. The risk of stroke increases in patients older than 60 years, with a history of hypertension or diabetes mellitus, and whose symptoms last > 10 minutes or are associated with weakness or speech impairments. Even though hypertension is a risk factor for TIA (A) or ischemic stroke, and can cause cerebral impairment, the AHA/ASA recommends permissive hypertension if the patient is not receiving thrombolytics. In such cases, blood pressure should be acutely lowered only if it remains persistently > 220/120 mm Hg with a goal of reducing by 15% within 24 hours. This patient's symptoms have resolved (B), so he is not a candidate for thrombolysis. Unfractionated and low molecular weight heparin (E) have not been shown to improve outcomes in stroke patients and increase the risk for intracerebral hemorrhage. Warfarin (D) is typically prescribed to patients with atrial fibrillation to reduce the risk of stroke from a thrombotic embolism. It should not be started until a full stroke workup has been performed.

Which of the following is most characteristic of a patient with idiopathic intracranial hypertension? Abnormal CSF chemistry analysis Opening pressure of 15 cm H2O on lumbar puncture Transient visual loss Unilateral papilledema

Correct Answer ( C ) Explanation: The exact pathophysiology of idiopathic intracranial hypertension (formerly referred to as pseudotumor cerebri or benign intracranial hypertension) is not well understood but suspected to result from an imbalance between the production and reabsorption of cerebrospinal fluid (CSF). It is most commonly seen in young, obese women of childbearing age. Additional risk factors include the use of oral contraceptives, anabolic steroids, tetracyclines and vitamin A. Headache is the predominant condition of the disease. The headache is usually generalized and made worse by maneuvers that impair cerebral venous return (e.g. Valsalva and bending forward). Visual complaints occur commonly and patients may have transient visual disturbances several times a day. After these episodes, periods of visual loss can occur. These episodes likely occur secondary to ischemia of the visual pathways. On physical examination, the patient may have an enlarged blind spot and loss of peripheral vision. Rarely, a sixth nerve palsy occurs. Abnormal CSF chemistry analysis (A) is not typical for idiopathic intracranial hypertension. Most often the analysis is normal, although protein can be low. The opening pressure on lumbar puncture is elevated due to the intracranial hypertension. An opening pressure of 20 cm of H2O would be considered the upper limits of normal. In patients who are obese, a pressure >25 cm H2O is considered abnormal and in non-obese patients, a pressure of >20 cm H2O. Therefore a pressure of 15 cm H2O (B) would be considered normal. Papilledema (D) is often seen, but occurs bilaterally because of the elevated intracranial pressure.

An 18-year-old college student presents with altered mental status and fever. The patient's roommate reports that the patient complained of headache earlier in the day. The patient is started on antibiotics and sent for a CT scan which is normal. A lumbar puncture is performed and the gram stain demonstrates gram negative diplococci. Which of the following should be administered to the patient's close contacts? Amoxicillin Doxycycline Rifampin Trimethoprim-sulfamethoxazole

Correct Answer ( C ) Explanation: The patient described has meningitis caused by Neisseria meningitides identified on the gram stain (Gram negative dipplococci). Antibiotics for the patient should not be delayed and should include ceftriaxone and vancomycin. Neisseria meningitides is highly contagious and may be responsible for outbreaks in close quarters including college dorms and army barracks. Given its contagious nature, antibiotic prophylaxis is indicated for close contacts of the patient including those in contact with secretions as well as members of the same household, daycare center or nursery school. Healthcare workers with close contact with the patient's secretions should also receive prophylaxis. Rifampin may be administered at a dose of 10 mg/kg to a maximum of 600 mg every 12 hours for four doses. Other alternatives include intramuscular ceftriaxone and ciprofloxacin. Antibiotic prophylaxis is also indicated for meningitis caused by Hemophilus influenzae. Amoxicillin (A), doxycycline (B) and trimethoprim-sulfamethoxazole (D) do not have adequate activity against Neisseria and should not be used for prophylaxis. The administration of the meningitis vaccine may also be considered in both epidemic and sporadic outbreaks as an additional adjunct to prophylaxis.

A 13-year-old boy on chemotherapy for acute lymphoblastic leukemia presents with progressive lower back pain for 2 weeks. Per mom, he has had subjective fevers and a "bulge in his lower back" that is warm to touch. He is currently afebrile and has no focal neurologic deficits. An MRI is obtained as seen in the image above. Which of the following regarding this patient's condition is true? An appropriate antibiotic regimen is cefepime and metronidazole Direct extension of skin and soft tissue infections is the most common cause Erythrocyte sedimentation rate is a sensitive screening tool Most patients present with back pain, fever, and a focal neurologic deficit

Correct Answer ( C ) Explanation: The patient has a posterior epidural abscess with adjacent osteomyelitis and myositis. The most common cause of an epidural abscess is hematogenous spread of infection, not direct extension from skin or soft tissue infection. Major risk factors include diabetes, intravenous drug abuse, chronic renal failure, alcoholism, and immunosuppression. The most common organism involved is Staphylococcus aureus. Other organisms include streptococci, anaerobes, gram-negative bacilli, and Pseudomonas aeruginosa. Patients present with localized back pain with tenderness to percussion. Fevers are common (reported in up to 75% of patients). MRI is the diagnostic modality of choice, but erythrocyte sedimentation rate is a sensitive marker and may be used in conjunction with plain radiographs to screen for infectious spinal disease. Vancomycin is an important component of the antibiotic regimen because it covers methicillin-resistant Staphylococcus aureus (MRSA) and is usually combined with a 3rd-generation cephalosporin (A). Cefepime is a 4th-generation cephalosporin with pseudomonas coverage. Metronidazole or clindamycin can be added for anaerobic coverage. While the classic triad consists of back pain, fever, and neurologic deficits (D), only a small proportion of patients actually have all three components at presentation. The most common cause of an epidural abscess is hematogenous spread of infection, not direct extension from skin or soft tissue infection (B).

A 35-year-old man is administered IV metoclopramide for migraine abortive therapy. One hour later the patient is noted to have the following activity seen in the video above. Which of the following medications should be administered to the patient? Bromocriptine Dantrolene Diphenhydramine Muscarine

Correct Answer ( C ) Explanation: The patient is having an acute dystonic reaction secondary to metoclopramide. Acute dystonic reactions manifest as involuntary spasms that often involve the face, neck, back, or limb muscles. Although dystonic reactions are occasionally dose related, these reactions are more often idiosyncratic and unpredictable. Dystonic reactions are commonly associated with typical antipsychotics (haloperidol, droperidol, chlorpromazine) but can also occur, albeit less frequently, with atypical antipsychotics (clozapine, olanzapine) and other antidopaminergic drugs like metoclopramide. They reportedly arise from a drug-induced alteration of dopaminergic-cholinergic balance in the nigrostriatum (i.e., basal ganglia). Most drugs produce dystonic reactions by nigrostriatal dopamine D2 receptor blockade, which leads to an excess of striatal cholinergic output. High-potency D2 receptor antagonists are most likely to produce an acute dystonic reaction.Acute treatment consists of administration of anticholinergic agents such as diphenhydramine or benztropine. Patients should continue therapy for at least 48 hours to prevent recurrence and be referred back to their prescribing physician for medication adjustment.

A 22-year-old man presents to the ED with sustained spasm of his neck to the right. Which of the following drugs is he most likely to be taking? Benztropine Cocaine Haloperidol Ziprasidone

Correct Answer ( C ) Explanation: This man is having a dystonic reaction, which is a common side effect of haloperidol and other typical antipsychotic drugs. Haloperidol is a high-potency antipsychotic that blocks dopamine-2 receptors at the basal ganglia, which can lead to acute dystonia shortly after drug initiation (50% occur within 48 hours, 90% within five days) and sustained movement disorders (parkinsonism and tardive dyskinesia) with prolonged use. Symptoms of acute dystonia include intermittent, involuntary, and uncoordinated hyperkinetic movements most often affecting the tongue, face, neck, trunk, or extremities. Treatment is with IM or IV benztropine or diphenhydramine, and recovery is rapid after medication administration. Benztropine (A) has both anticholinergic and antihistamine activity and is commonly used for the treatment of movement disorders. Although cocaine (B) does not typically cause dystonia, it can increase the risk for dystonic reactions and is associated with choreoathetoid movements that are referred to as crack dancing. Ziprasidone (D) is an atypical antipsychotic drug that is much less likely to cause dystonia and other extrapyramidal symptoms.

A 32-year-old woman complains of back pain and difficulty urinating for five days. The pain is located in her lumbar spine and is constant and dull. She says she feels the urge to urinate every five minutes, but only a small amount of urine is expressed. On physical exam, her vital signs are T 38.4°C, BP 110/72 mm Hg, HR 115 bpm. Cardiac exam reveals a 2/6 systolic murmur. Lung exam is normal. Abdominal exam reveals a large, tender suprapubic mass. Musculoskeletal exam reveals tenderness over her lumbar spine and track marks consistent with IV drug use in her left antecubital fossa. Neurological exam reveals 4/5 strength in her bilateral lower extremities. What is the most likely cause of her symptoms? Bladder cancer Necrotic uterine leiomyoma Spinal epidural abscess Tubo-ovarian abscess

Correct Answer ( C ) Explanation: This patient has a spinal epidural abscess causing cauda equina syndrome. Urinary retention with or without overflow incontinence is the most common finding of cauda equina syndrome. Spinal epidural abscesses classically present with back pain, fever, and neurologic deficits. In this case, it is urinary retention. Some risk factors for epidural abscess include injection drug abuse, immunocompromised state, recent spinal surgery, and cancer. This patient has a cardiac murmur and track marks on her arm confirming her history of IV drug abuse. This patient's suprapubic mass is due to urinary retention, not a bladder malignancy. Bladder cancer (A) is most commonly found in older adults with a history of smoking and an occupational exposure to certain chemicals. A necrotic uterine leiomyoma (B) can present with fever and pelvic pain, but is usually the result of a rapidly growing leiomyoma or as a complication of uterine artery embolization. It is not associated with neurologic deficits. A tubo-ovarian abscess (D) is part of the spectrum of pelvic inflammatory infections. Lower abdominal pain and fever are frequently present. In addition, there is unilateral adnexal tenderness.

A 64-year-old man complains of pain and paresthesias in his right hand intermittently for several weeks.He works in a factory putting together electronics.On exam, he has decreased sensation of his right 1st through 4th digits and an atrophied thenar eminence.What test in the ED will help diagnose his condition? CT scan of the head Electromyelography (EMG) Percuss the right volar wrist Urine drug screen

Correct Answer ( C ) Explanation: This patient has median mononeuropathy, also known as carpal tunnel syndrome, a compression neuropathy of the median nerve as it traverses under the flexor retinaculum at the wrist.The median nerve provides sensation primarily to the palmar aspect of the 1st, 2nd, 3rd, andradial side of the 4th.When it is compressed, the patient experiences pain, paresthesias, and numbness in that distribution.The Tinel's test is performed by lightly tapping the volar surface of the wrist over the median nerve.This should elicit a sensation of tingling or pins and needles in the distribution of the median nerve. Carpal tunnel syndrome is first treated with wrist splinting and initiation of a more ergonomic work environment. NSAIDs may also be helpful. If symptoms do not improve, the patient should be referred to a hand specialist who may elect to perform a carpal tunnel release procedure CT scan of the head (A) would be useful if there issuspicion that this patients symptoms are from a central process such as an acute stroke. However, his numbness is in a peripheral nerve distribution and not dermatomal.An EMG (B) is used most commonly by neurologists to confirm damage to peripheral nerves. It may be used for carpal tunnel syndrome if the symptoms do not resolve with conservative management.A variety of heavy metals are associated with a peripheral neuropathy (lead, mercury), but these require special serologic testing, not a urine drug screen (D).The patient is at risk for occupational exposures because he works in a factory; however, most heavy metal poisonings are associated with other symptoms.

A 73-year-old woman with a history of hypertension presents with a unilateral headache for three weeks. She states that she has a throbbing pain at her right temple and has pain in her jaw with opening and closing. The vision in her right eye has worsened over the previous day. Her blood pressure is 173/100 mm Hg. What treatment is indicated? Carbamazepine Labetalol Methylprednisolone Non-contrast head CT scan

Correct Answer ( C ) Explanation: This patient presents with a unilateral, subacute headache with associated jaw claudication and vision change; symptoms consistent with temporal arteritis. Temporal arteritis or giant cell arteritis is a systemic inflammatory process of large and medium-size arteries. The most commonly involved vessels are the ophthalmic vessels and the extracranial branches of the aortic arch. The disease typically affects patients over 70 years of age and is more common in women than in men. Patients present with a subacute headache that is throbbing in nature and may be present for weeks to months. Often, patients will have symptoms for more than 2 months. Patients may also report jaw claudication secondary to vascular insufficiency of the masseter and temporalis muscles. Physical examination may reveal tenderness over the temporal artery. Systemic symptoms may also be present including fever, joint pains, and weight loss. Diagnostic testing in the Emergency Department generally begins with an erythrocyte sedimentation rate (ESR) with a cutoff of 50 mm/hour although the level may be >100 mm/hour. However, the ESR will be normal in 10-25% of patients. The gold standard diagnostic test is a temporal artery biopsy. In patients with a high-clinical likelihood of temporal arteritis, treatment should be initiated regardless of initial diagnostic testing as delay can lead to permanent visual loss. Prednisone should be started at 60 - 120 mg/day. If vision loss is present, methylprednisolone is the recommended medication. Carbamazepine (A) is the treatment of choice for trigeminal neuralgia, not temporal arteritis. The patient does not present with symptoms consistent with hypertensive emergency requiring emergent antihypertensive treatment with labetalol (B). A non-contrast head CT scan (D) is not helpful in temporal arteritis as the disease does not involve the intracranial contents.

A 20-year-old woman presents with weakness in her left wrist. She states that she fell asleep in a chair after a night of heavy drinking. On physical examination, she is unable to extend her wrist. What nerve is compromised? Axillary nerve Median nerve Radial nerve Ulnar nerve

Correct Answer ( C ) Explanation: This patient presents with a wrist drop caused by radial neuropathy also called "Saturday Night Palsy." The radial nerve arises from the C5-T1 roots. It controls extension of the fingers, thumb, wrist and elbow. Symptoms of radial neuropathy depend on the location of compression. Compression in the axilla (typically from improper use of crutches) causes weakness of extension at the elbow, wrist and fingers. More typically, the nerve is compressed between the humeral shaft and another hard surface and results in weakness with extension at the wrist and fingers. This typically occurs from deep sleep (often secondary to inebriation). About 90% of radial nerve mononeuropathies that occur during sleep, coma or anesthesia recover within 6-8 weeks. Patients should be placed in a wrist splint with 60 degrees of dorsiflexion to prevent atrophy and contractures. The axillary nerve (A) causes weakness of the shoulder (deltoid) and decreased sensation over the shoulder. Median nerve neuropathy (B) causes numbness in the 1st, 2nd and 3rd digits. Ulnar nerve (D) neuropathy symptoms depend on location but do not cause weakness with extension at the wrist.

A 22-year-old man presents with upper extremity weakness. He notes that he was drinking heavily last night and when he woke up this morning he was unable to move his hand. On examination, he has normal strength in the biceps and triceps but weakness of the wrist extensors, finger extensors, and brachioradialis. Injury to what nerve accounts for his symptoms? Median nerve Musculocutaneous nerve Radial nerve Ulnar nerve

Correct Answer ( C ) Explanation: This patient presents with findings consistent with a radial nerve palsy. The radial nerve is formed from nerve roots C6 through C8. It runs adjacent to the humerus in the spiral groove of the upper arm. Compression in this area can result in a radial mononeuropathy. This occurs more frequently in intoxicated patients resulting in the term "Saturday night palsy". On examination, the patient will have normal strength in the triceps as the radial nerve gives off the branch to the triceps muscle prior to wrapping around the humerus at the spiral groove. There will be weakness of the wrist extensors, resulting in a wrist drop, as well as weakness of the finger extensors and brachioradialis muscle. Sensory deficits will follow the radial nerve distribution which includes the radial aspect of the dorsum of the hand. Diagnosis is based on clinical findings. Treatment includes splinting the wrist, physical therapy, and pain management as needed. The median nerve (A) is responsible for the motor function of the flexor muscles of the anterior forearm compartment (except for the flexor carpi ulnaris and part of the flexor digitorum profundus), the thenar muscles and the lateral two lumbricals. Compression of the median nerve as it passes through the carpal tunnel of the wrist will result in pain and paresthesias in the distribution of the median nerve. The musculocutaneous nerve (B) innervates the coracobrachialis, the biceps brachii and the brachialis. Injury to this nerve will result in weakness of elbow flexion with associated sensory loss over the lateral forearm. Ulnar (D) neuropathy at the elbow is the second most common compression neuropathy and results in sensory loss and paresthesias over the fourth and fifth digits as well as weakness with finger and wrist flexion.

A 33-year-old man presents with a seizure lasting for five minutes. EMS administered 2 mg of lorazepam with cessation of seizure activity. On presentation, the patient is confused. The patient's medication list includes metoprolol and isoniazid. During the evaluation, he has another seizure lasting for 10 minutes and then a third seizure lasting for another 10 minutes. What adjunctive therapy should be given? Folic acid Phenobarbital Pyridoxine Sodium bicarbonate

Correct Answer ( C ) Explanation: This patient presents with status epilepticus and is taking isoniazid (INH). In patients with INH on the medication list and status epilepticus, empiric pyridoxine should be administered. Status epilepticus is defined as recurrent seizures without return of normal mental status in between episodes or for a prolonged duration. First line treatment for status is the same as for any seizure - benzodiazepines. Failure of benzodiazepines should prompt consideration of alternate medications including barbiturates, levitircetam, phenytoin and other antiepileptic medications. Additionally, a history of specific toxins should be obtained as this may guide management. In this case, the history of INH use should prompt consideration of pyridoxine treatment; the antidote for an INH overdose. Folic acid (A) is not a status epilepticus treatment. Phenobarbital (B) is a reasonable second line seizure medication for status epilepticus if no specific toxins are present. Sodium bicarbonate (D) is useful in the treatment of aspirin and tricyclic antidepressant induced status epilepticus.

An 82-year-old man presents with slurred speech and unilateral arm weakness that has resolved. His non-contrast head CT is negative and he is given aspirin. Which of the following is the most appropriate management plan? Discharge home with PCP follow-up Low molecular weight heparin MRI and MRA of the head and neck Transcranial dopplers

Correct Answer ( C ) Explanation: This patients symptoms are concerning for a transient ischemic attack (TIA), neurologic deficits that occur and then resolve within 24 hours. Most patients with a TIA will resolve within one hour (the previous definition). Patients who have a TIA are at high risk for stroke in the period immediately following a TIA and therefore further testing is warranted in a timely fashion. In most cases, patients should be admitted to an observation unit or hospital in order to rule out any reversible causes of the TIA. Testing includes vascular imaging to identify any critical stenosis. The MRI will also identify the subset of patients with TIA symptoms who ultimately have an infarct on MRI. If MRA is not available, carotid dopplers are an acceptable alternative. Once the vasculature has been evaluated, patients may be discharged with outpatient follow-up. Routine testing that often occurs during the observation period includes cardiac telemetry for dysrhythmia (particularly atrial fibrillation) and echocardiography to evaluate for valvular lesions or intracardiac thrombus. Without further testing, it is inappropriate to discharge home with PCP follow-up (A). Some data in Europe suggest that referral to a stroke neurologist for prompt follow-up and coordinated care may be a safe alternative. However, in the US, the vasculature is typically exonerated prior to discharge. Low molecular weight heparin (B) is not standard treatment for TIA. In cases where atrial fibrillation is identified, patients will begin anticoagulation in the absence of contraindications. Transcranial dopplers (D) are sometimes used to evaluate the posterior circulation. However, this patients symptoms are not consistent with a posterior stroke and therefore are not indicated. Furthermore, the sensitivity of transcranial dopplers is poor for the identification of critical vascular stenosis.

A 61-year-old man presents with intermittent shock-like spasms of pain in his right cheek. The pain only lasts a few seconds at a time but recurs frequently. He notes that shaving each morning causes intense spasms of pain, as does chewing. What is the first-line treatment for his likely diagnosis? Baclofen Botulinum injections Carbamazepine Diazepam

Correct Answer ( C ) Explanation: Trigeminal neuralgia is characterized by unilateral, shock-like paroxysms of pain in distributions of one or more divisions of the trigeminal nerve. It is often the result of vascular compression of the trigeminal nerve root. It is more common in women than men and typically affects patients over 50 years of age. The pain can be triggered by such things as chewing, shaving, light touch or exposure to hot or cold temperatures. Diagnosis is made clinically based on history and exclusion of other possible sources of pain, such as an odontogenic infection, temporomandibular joint disease, and herpes zoster. The first-line treatment of choice for trigeminal neuralgia is carbamazepine. The initial dosage is 100 mg twice daily. This can be increased up to 1200 mg/day as needed for symptom control. Patients should be monitored periodically with laboratory studies to rule out potential hematologic or hepatic side effects. Baclofen (A) has been used in the treatment of trigeminal neuralgia although there is limited evidence of its efficacy. Botulinum injections (B) may be useful in patients with pain that is refractory to medical options. Diazepam (D) is not used in the management of trigeminal neuralgia.

Which of the following patients require a CT scan prior to performing a lumbar puncture for suspected meningitis? A 10-day-old infant with temperature of 102.9°F who appears irritable A 19-year-old college student with a temperature of 104.3°F and severe meningismus A 59-year-old man with temperature of 102.3°F, a severe throbbing headache, vomiting, and meningismus A 65-year-old woman with known lung cancer who presents with a temperature of 100.9°F, headache, and meningismus

Correct Answer ( D ) Explanation: A CT scan should be obtained prior to performing a lumbar puncture (LP) when there is concern for brain herniation in a patient with a possible space-occupying lesion. While the overall incidence of this complication is thought to be extremely low, CT prior to LP is recommended for the following patients with suspected meningitis. A 10-day-old infant with temperature of 102.9°F and appears irritable (A), a 19-year-old college student with a temperature of 104.3°F and severe meningismus (B), a 59 year-old man with a severe headache and vomiting (C) do not meet any criteria requiring a CT scan prior to LP.

A 21-year-old man presents with a headache. What feature should raise the concern for a subarachnoid hemorrhage? Age <40 Fever History of intravenous drug use Sudden onset of headache

Correct Answer ( D ) Explanation: A sudden onset of headache should raise concern for subarachnoid hemorrhage (SAH). SAH is a life-threatening disorder that involves extravasation of blood into the subarachnoid space. It accounts for 10% of all strokes. SAH can either be traumatic or spontaneous. Spontaneous SAH is associated with intracranial aneurysms about 80% of the time. Patients typically present with a sudden onset of severe headache that is classically described as "the worst headache of their life." The headache of SAH is often associated with nausea and vomiting (75%) and may have concomitant neck stiffness (25%) and seizures (17%). Meningismsus is present in half of patients. The workup of a patient with a suspicion of a subarachnoid hemorrhage typically involves a noncontrast head CT followed by a lumbar puncture if imaging is unremarkable. Although the ability of CT to detect SAH in patients presenting early in the course of disease has improved greatly over the last decade, lumbar puncture is still recommended to rule out the disorder.

A previously healthy 48-year-old man presents with fever, headache and signs of meningeal irritation. He has no significant past medical history. He has no rash and has not had any recent medical procedures. Which of the following is the most common bacteria responsible for the suspected diagnosis? Haemophilus influenzae Neisseria meningitidis Staphylococcus aureus Streptococcus pneumoniae

Correct Answer ( D ) Explanation: Adult bacterial meningitis has several microbiologic etiologies. The most common cause is Streptococcus pneumoniae (30-60% of cases) for all age categories. The second most common bacteria is Neisseria meningitidis (10-35%), which more commonly affects older children and young adults, and more commonly occurs is associated with a petechial rash. Haemophilus influenzae is also causative (<5%), and occurs most commonly in those with recent neurosurgical procedures or possible CSF leaks. Listeria monocytogenes (5-10%) and Gram-negative rods (1-10%) are also causative and occur more frequently in elderly patients or those with alcoholism, immunosuppression, cancer and recent medical or neurosurgical procedures. Empiric treatment in healthy adults is ceftriaxone plus vancomycin. Consider ampicillin if the patient is older than 50 years or has an alcohol abuse history in order to cover Listeria monocytogenes. Haemophilus influenzae (A) is not a common source in this age group nor in those without recent neurosurgical procedures. Neisseria meningitidis (B) is less common in this age group, and is less common overall when compared to Streptococcus pneumoniae. Staphylococcus aureus (C) is seen in patients with indwelling catheters, recent head trauma and recent neurosurgical procedures.

Which of the following cerebral spinal fluid results is most consistent with a diagnosis of bacterial meningitis? Glucose 60 mg/dL Glucose 90 mg/dL Protein 30 mg/dL Protein 90 mg/dL

Correct Answer ( D ) Explanation: An elevated CSF protein of 90 mg/dL is consistent with a diagnosis of bacterial meningitis. Normal adult CSF protein levels are between 15 and 45 mg/dL. An elevated CSF protein is indicative of meningitis (from all causes), encephalitis, subarachnoid hemorrhage, vasculitis, demyelinating disease and malignancy. There are several types of meningitis including bacterial, viral, fungal and parasitic. Bacterial meningitis is caused by multiple pathogens including S. pneumoniae, N. meningitides, and Listeria monocytogenes. The likelihood of each pathogen varies based on patient age and comorbid state. Signs and symptoms associated with bacterial meningitis include fever, vomiting, meningismus, headache, photophobia, malaise, lethargy, altered mental status and seizures. Significant morbidity and mortality is associated with bacterial meningitis. Bacterial meningitis is diagnosed via lumbar puncture with cerebral spinal fluid analysis. Specific CSF characteristics are noted and laboratory tests are sent in order to diagnose meningitis. The initial measurement is an opening pressure, which should be measured in the lateral decubitus position. Next, the turbidity of the fluid should be noted as it is obtained. Cell count and differential are often abnormal in bacterial meningitis. Normal CSF contains no more than 5 WBCs/hpf and no more than one PMN/hpf. WBC counts of greater than 500 with >90% PMNs are suggestive of bacterial meningitis. Gram stain yields the causative organism up to 80% of the time. CSF glucose is generally low in bacterial meningitis. The CSF-to-serum glucose ratio is 0.4 in normal individuals. Generally CSF glucose levels of 50-100 mg/dL are considered normal (A and B). Normal adult CSF protein levels are between 15 and 45 mg/dL (C). In the case of bacterial meningitis, CSF protein levels are elevated.

Which of the following is the most sensitive finding in cauda equina syndrome? Back pain Saddle anesthesia Urinary incontinence Urinary retention

Correct Answer ( D ) Explanation: Cauda equina syndrome refers to a characteristic pattern of neuromuscular and urogenital symptoms resulting from the simultaneous compression of multiple lumbosacral nerve roots below the level of the conus medullaris. Midline rupture of the L4/L5 disk is the most common cause. Other causes include tumors, trauma, or spinal epidural abscesses. The presence of urinary retention is the most sensitive finding (90% sensitivity). A postvoid residual of greater than 100-200 ml should alert the emergency physician to the likelihood of this syndrome in the right clinical setting. Lower motor neuron findings may be present, in contrast to upper motor neuron findings, which suggest conus medullaris syndrome (a true cord syndrome). MRI is the diagnostic modality of choice. Cauda equina syndrome is a neurosurgical emergency that requires operative decompression.

Which of the following requires chemoprophylaxis for meningitis exposure? Breastfeeding mother of an infant with E. coli meningitis Classmates of a 7-year-old with S. pneumoniae meningitis Respiratory therapist (vaccinated) for a 10-year-old with H. influenzae meningitis Roommate of a 19-year-old with N. meningitidis

Correct Answer ( D ) Explanation: Chemoprophylaxis with rifampin is recommended for all high-risk contacts of patients with documented meningitis caused by N. meningitidis. High-risk contacts include household members, school contacts in the prior seven days, and patients with direct exposure to secretions. Patients given chemoprophylaxis should be advised to seek medical treatment immediately if they develop signs or symptoms of meningitis. E. coli (A) rarely causes meningitis outside of the newborn period. Chemoprophylaxis is not recommended for these contacts. S. pneumoniae (B) meningitis does not require chemoprophylaxis for contacts. H. influenzae (C) meningitis is rare now that the H. influenzae vaccine is prevalent in the population. Vaccinated health care workers do not require chemoprophylaxis. However, chemoprophylaxis is recommended in confirmed cases of H. influenzae meningitis where the exposed person has not been vaccinated.

A 35-year-old woman with a history of migraines and polycystic kidney disease presents to the Emergency Department with a severe, diffuse headache. The onset was abrupt approximately one hour prior to arrival. Her vital signs are within normal limits. She has photophobia and phonophobia, as well as pain with extraocular movements. Which of the following is the most likely diagnosis? Classic migraine Idiopathic intracranial hypertension Meningitis Subarachnoid hemorrhage

Correct Answer ( D ) Explanation: Subarachnoid hemorrhage (SAH) accounts for approximately one-third of all hemorrhagic strokes. Atraumatic SAH is due to ruptured aneurysms. A history of polycystic kidney disease is a risk factor for the development of berry aneurysms which may spontaneously rupture and cause a subarachnoid hemorrhage. Other medical conditions associated with SAH include Marfan syndrome, coarctation of the aorta and fibromuscular dysplasia. Classic symptoms include an abrupt "thunderclap" headache that is maximal in severity at onset. Patients often have signs of meningeal irritation secondary to blood in the subarachnoid space. These include nuchal rigidity, painful extraocular movements, photophobia, and a positive Brudzinski or Kernig sign. Subarachnoid blood appears white on noncontrast head CT and most often appears in the cerebral cisterns within which lies the vessels that compose the circle of Willis. Management includes supportive care, including airway management as needed, nimodipine, and neurosurgical consultation.

A young woman presents with bilateral headache. She describes it as a band-like pressure from her forehead to neck, feeling as if her "head is in a vice." She denies prodromal symptoms, nausea and photophobia. Cervical spine range-of-motion and skull palpation do not reproduce the headache. Which of the following is the most likely diagnosis? Cluster headache Migraine without aura Occipital headache Tension-type headache

Correct Answer ( D ) Explanation: Tension-type headache (TTHA) is the most common type of recurring headache other than migraine and the most common type overall. Tension headaches present as a bilateral headache with pain in the frontal and occipital regions in a bandlike distribution. Headaches are exacerbated by stress, fatigue, glare, or noise. It also often involves contraction of the scalp and posterior neck muscles, with a normal neurologic exam. Treatment is with aspirin, NSAIDs, and ergotamines. Unilateral, throbbing headache with nausea and photophobia is much more indicative of migraine headache (B)​. These symptoms are absent in tension-type headaches. Occipital headaches (C), sometimes called occipital neuralgia, C2 neuralgia, C2 headaches, are characterized by sharp shooting electric-like occipital pain that radiates cephalad, retroorbitally or in the distribution of the greater and lesser occipital nerves. This pain typically worsens with upper cervical spine range-of-motion. A common exam finding of occipital headaches is reproduction of radiating pain during palpation of the occipital nerves as they exit the ligamentum nuchae at the base of the skull. Cluster headache (A) has a predilection for males and is associated with excrutiating periorbital pain, ipsilateral conjunctival injection, lacrimation, and nasal discharge

Which of the following is more consistent with the diagnosis of encephalitis rather than meningitis? Fever Headache Neck stiffness New psychiatric symptom

Correct Answer ( D ) Explanation: The clinical distinction between encephalitis and meningitis is typically characterized by the presence of a distinct neurologic abnormality in encephalitis. Encephalitis should, be considered in patients presenting with the following clinical features, alone or in combination: new psychiatric symptoms, cognitive deficits (aphasia, amnesia, acute confusional state), seizures, and movement disorders. Often, there are symptoms of meningeal involvement as well, such as a headache, photophobia, and fever. There is overlap between encephalitis and meningitis. When this occurs, the condition is referred to as meningoencephalitis

A 19-year-old woman at 34 weeks gestation presents with a one-day history of progressive left-sided facial weakness. She notes a recent history of an upper respiratory infection. On exam, she is unable to raise her left eyebrow, close her left eye, or smile on her left side. She also reports decreased taste sensation. Her neurologic exam is otherwise unremarkable. Which of the following complications is most commonly associated with the patient's diagnosis? Cavernous sinus thrombosis Descending paralysis Hearing loss Keratitis

Correct Answer ( D ) Explanation: The patient has unilateral facial nerve palsy with no other focal neurologic or systemic findings. Although the differential diagnosis for facial nerve paralysis is extensive and includes infectious, traumatic, and neoplastic etiologies, the most common diagnosis is idiopathic Bell's palsy. Bell's palsy is characterized by the sudden onset of facial nerve paralysis with symptoms usually peaking by 48 hours. Some 60% of patients have a viral prodrome. In making the diagnosis, careful consideration must be given to the motor function of the forehead; sparing of this region would be characteristic of a central, and not peripheral, etiology. Because the orbicularis oculi muscles are involved, this results in incomplete closure of the eyelids on the affected side and leads to corneal exposure keratitis. It is important to prescribe lubricating eye drops, to keep the corneal epithelium from breaking down, and to patch the affected eye at nighttime. Cavernous sinus thrombosis (A) is a cerebral venous thrombosis that presents with orbital pain, proptosis, and oculomotor palsies. The cavernous sinus contains CN III, IV, V1, V2, and VI. Resultant facial sensory nerve loss is common. Descending paralysis (B) is not a feature of Bell's palsy. Botulism is associated with facial muscle weakness and descending paralysis. Bell's palsy is associated with abnormally acute hearing (hyperacusis) (C), not the loss of hearing. A similar facial nerve paralysis, Ramsay Hunt syndrome (herpes zoster oticus), is characterized by unilateral facial paralysis, a herpetiform vesicular eruption, and vestibulocochlear dysfunction with the possibility of complete hearing loss. In addition, mastoiditis can progress to cause a facial nerve palsy, but it is not a complication of idiopathic Bell's palsy.

A 45-year-old woman presents complaining of one day of vision loss in her left eye associated with a left retro-orbital headache. She denies trauma or preceding illness. On exam, her vital signs are T 36.8ºC, BP 112/80 mm Hg, HR 74 bpm. Visual acuity is 20/20 in the right eye, 20/400 in the left eye, and 20/40 using both eyes. She has a left-sided afferent pupillary defect. Visual field testing reveals left-sided central vision loss. Which of the following is most accurate regarding her condition? Strict blood glucose control reduces the likelihood for future recurrences The diagnosis is confirmed by measurement of an intraocular pressure > 20 The diagnosis is made by magnetic resonance venography (MRV) The syndrome is associated with multiple sclerosis Therapy should be aimed at removing the acute obstruction of the ophthalmic artery

Correct Answer ( D ) Explanation: The patient's exam is consistent with optic neuritis, an acute monocular loss of vision caused by focal demyelination of the optic nerve. Most cases are retrobulbar and do not involve any visible changes on fundoscopy (especially during an acute episode). But, on occasion and with more protracted cases, optic disk pallor may be seen. An afferent pupillary defect, however, is always present. Approximately 30% of patients presenting with acute optic neuritis develop multiple sclerosis within 5 years. The classic clinical syndrome of MS consists of recurring episodes of neurologic symptoms that rapidly evolve over days and slowly resolve. Controlling blood glucose levels (A) has not been shown to influence the course of optic neuritis or multiple sclerosis. Optic neuritis is not associated with (B) increased (i.e., > 20) intraocular pressure. Magnetic resonance venography (MRV) (C) does not have a role in the diagnosis of optic neuritis or MS. However, a T2-weighted MRI can aid in the diagnosis by demonstrating lesions in the periventricular white matter—a finding that differentiates simple optic neuritis (where white matter changes are absent) from MS. Such an approach is critical in restoring vision in cases of central retinal artery occlusion (E), not optic neuritis.

Assuming that all of the following patients have returned to baseline and have normal glucose levels, which of the following patient needs further workup in the Emergency Department for first-time seizure? A 10-month-old with a one day temperature of 38.3°C; seizure lasted for 5 minutes A 15-month-old with a one day temperature of 38.0°C; seizure lasted for 10 minutes A 3-year-old with a one day temperature of 40.2°C; seizure lasted for 3 minutes An 8-year-old with a two day temperature of 38.8°C; seizure lasted for 5 minutes

Correct Answer ( D ) Explanation: This child had a complex febrile seizure. Complex febrile seizures have the following characteristics: duration > 15 minutes, focality, and age < 6 months or > 5 years without signs of serious infection. Moreover, this patient's seizure occurred after two days of fever which puts this patient at increased risk for a serious bacterial infection and possibly meningitis. The patient requires further workup, including CBC, blood cultures, urine analysis, urine culture, and CSF analysis with culture before the patient can be sent home. All such patients should be reevaluated within 24 hours to follow up on cultures and for repeat examination. Anticonvulsants are not recommended, and there is no evidence that antipyretics will prevent subsequent febrile seizures.

A 16-year-old girl presents to the ED complaining that she cannot walk up the steps. She has been well recently except for an episode of gastroenteritis two weeks ago. On exam, she has decreased strength in her bilateral lower extremities with absent patellar and ankle jerk reflexes. What is the most important next test to perform? Electromyelography Lumbar puncture MRI lumbar spine Vital capacity

Correct Answer ( D ) Explanation: This patient has Guillain-Barré syndrome (GBS), an immune-mediated peripheral neuropathy caused by myelin sheath destruction. It is often preceded by a viral illness, infection with Campylobacter jejuni, or vaccination. In the classic presentation of GBS, a viral illness is followed by an ascending symmetric weakness or paralysis with decreased or absent deep tendon reflexes. The most serious complication of GBS is respiratory failure from diaphragmatic weakness. Measuring the vital capacity or negative inspiratory force (NIF) is critical to assessing respiratory effort. These easily repeated tests predict developing diaphragmatic weakness and the need for prophylactic intubation. GBS is treated with supportive care, IV immune globulin, and possibly plasmapheresis. All patients with suspected GBS should be admitted to an intensive care unit. Electromyelography (A) (EMG) measures the action potential of a nerve and its muscle unit and provides information on the innervation status of a muscle, less likely to be abnormal in acute demyelination without axonal loss. EMG readings will show slowed action potentials in GBS because of demyelination. Although it is useful to distinguish a neuropathic etiology of weakness, it is not diagnostic of GBS, nor does it predict potential complications such as respiratory failure. Lumbar puncture (B) is performed to augment the diagnosis of GBS, but is not the test of choice; it has no predictive value. CSF will often show few WBCs, with a monocytic predominance and increased protein. Although MRI (C) of the lumbar spine may be performed to rule out a cord-compressing, space-occupying lesion that could be causing bilateral weakness, it provides no other specific information and will be negative in cases of GBS.

A 73-year-old man with a history of a spontaneous intracranial hemorrhage 2 years ago presents with 45 minutes of slurred speech and left-sided weakness. His vitals are T 98.7°F, HR 94, BP 173/97, RR 18, and finger stick 102. A non-contrast head CT shows no acute bleeding and no focal mass. What is the appropriate management for the patient? Administer parenteral medications to control the blood pressure, administer thrombolytics, and admit to a stroke unit Administer thrombolytics and admit to a stroke unit Hold thrombolytics and admit to a regular inpatient bed Hold thrombolytics and admit to a stroke unit

Correct Answer ( D ) Explanation: This patient has signs and symptoms concerning for an acute ischemic cerbrovascular accident (CVA). Alteplase (tPA), a thrombolytic agent, is recommended for use in patients presenting with acute ischemic stroke within 180 minutes of the onset of symptoms. tPA use in acute ischemic stroke also has a number of contraindications.

A 12-year-old boy presents with facial weakness for two hours. He states he woke up this morning and noticed that the right side of his face "doesn't move the right way." Examination reveals a right facial droop of the upper and lower face. He is unable to fully close his right eye. Examination is otherwise unremarkable. What management should be pursued? CT scan of the head Intravenous thrombolytics Lumbar puncture Prednisone

Correct Answer ( D ) Explanation: This patient presents with a peripheral facial nerve (CN VII) palsy or Bell's palsy, which should be managed with oral steroids (prednisone). Bell's palsy is an idiopathic, peripheral facial nerve palsy. The facial nerve innervates the muscles of the scalp, external ear and the muscles of facial expression. Additionally, the sensory portion supplies the anterior two thirds of the tongue. Patients will typically present with complaints of weakness of the facial muscles including the mouth, eyelid and forehead. One of the critical issues is to ensure that the paralysis is, in fact, peripheral and not central. In central facial nerve palsy, the muscles of the upper part of the face (forehead in particular) are spared. This is due to the fact that the upper third of the face has dual innervation centrally from bilateral cerebral cortexes. Thus, in a patient who has sparing of the upper part of the face, further diagnostic workup should be performed to determine the cause of central facial nerve paralysis. Peripheral facial nerve palsy is treated with prednisone 1 mg/kg/day for 7-10 days. The role of antivirals (i.e. acyclovir) is controversial as it is unclear if herpes simplex virus 1 is a causative organism. Additionally, artificial tears and taping the affected eye closed at night are recommended to prevent drying out of the cornea and subsequent abrasions or ulcerations. CT scan of the head (A) does not play a role as the lesion in Bell's palsy is outside of the central nervous system. Bell's palsy is not part of a stroke syndrome and thus should not be treated with thrombolytics (B). Peripheral facial nerve palsy does not result from central nervous system infection obviating the need for a lumbar puncture (C).

A 45-year-old man with a history of alcohol abuse presents with numbness and weakness of the left hand. He states he slept on a bench last night and awoke this morning with the symptoms. Physical examination reveals decreased sensation over the first, second, and thirddigits and a wrist drop. What management is indicated? CT of the cervical spine MRI of the brain Noncontrast CT scan of the head Wrist splint and follow up with neurology

Correct Answer ( D ) Explanation: This patient presents with a radial nerve palsy likely caused by direct compression and should have a wrist splint placed and given follow up with a neurologist. The radial nerve arises from the C5-T1 roots, exits the brachial plexus and passes into the spiral groove behind the proximal humerus. It bifurcates at the antecubital fossa. This nerve controls extension of the fingers, thumb and wrist. Compression of the radial nerve in the axilla occurs secondary to the improper use of crutches or from an extended period of unconsciousness with prolonged compression. This pathology is typically called a Saturday night palsy. Patients with radial nerve palsy present with weakness in extension of the fingers and wrist. Spontaneous recovery often occurs but may take 2-4 months. During this time, the wrist should be splinted with 60 degrees of volar flexion in order to prevent atrophy and contractures. CT of the cervical spine (A), MRI of the brain (B) and CT scan of the head (C) are not indicated as this is a peripheral nerve injury.

A 63-year-old man with a history of alcoholism presents with fever, headache and altered mental status. What antibiotics should be administered? Ceftriaxone, ampicillin Vancomycin, cefazolin, ampicillin Vancomycin, ceftriaxone Vancomycin, ceftriaxone, ampicillin

Correct Answer ( D ) Explanation: This patient presents with symptoms concerning for meningitis and should immediately have antibiotics started covering the most likely pathogens. Bacterial meningitis typically presents with a rapidly progressive course. There are a number of causative organisms but the most common are Streptococcus pneumoniae, Neisseria meningitidis and Listeria monocytogenes. Streptococcus pneumoniae ​is the most common organism in all adults, while N. meningitidis is more likely to be encountered in younger adults. L. monocytogenes is more common at the extremes of age (neonates and the elderly) and in patients with immunosuppression and alcoholism. Patients may experience fever, headache, photophobia, neck pain, vomiting, altered mental status, lethargy and seizures. Kernig's and Brudzinski's signs (physical findings consistent with nuchal rigidity) are classic findings but are seen in less than 50% of adults. A non-contrast head CT should be performed in patients with suspected meningitis if they have signs of increased intracranial pressure or mass effect (focal neurologic findings, altered mental status or papilledema) and in those with significant immunocompromise (i.e. HIV/AIDS). The diagnosis of meningitis is made by analysis of the CSF after lumbar puncture. Antibiotics should not be withheld from patients while waiting for CT scan or lumbar puncture. While awaiting the results of CSF and blood cultures, broad-spectrum antibiotics covering the most likely pathogens should be started. Vancomycin is administered to cover resistant S. pneumoniae and a third generation cephalosporin (ceftriaxone) for N. meningitidis. Ampicillin should be given for patients who are at risk for L. monocytogenes. The combination of ceftriaxone and ampicillin (A) fails to cover patients for resistant S. pneumoniae. Cefazolin (B) is a first generation cephalosporin and will not cover N. meningitidis. Vancomycin and ceftriaxone (C) is adequate in patients not at risk for L. monocytogenes.

A woman presents with 30 minutes of double vision, vertigo, difficulty swallowing, and difficulty speaking. During her initial evaluation, these symptoms resolve and her neurologic exam returns to normal. Which of the following is the most appropriate diagnosis and the most likely affected artery? Embolic ischemia - anterior cerebral artery Thrombotic stroke - left anterior descending artery Transient ischemic attack - middle cerebral artery Transient ischemic attack - vertebrobasilar arteries

Correct Answer ( D ) Explanation: Transient ischemic attacks (TIAs) are characterized by an abrupt onset of focal neurologic symptoms lasting less than 24 hours and often lasting only five to 20 minutes. TIAs suggest impending thrombotic-ischemic stroke. Carotid pathology leads to TIAs demonstrated by hemiparesis, hemisensory, aphasia, confusion and transient monocular blindness. Vertebrobasilar insufficiency leads to TIAs marked by hemiplegia or quadriplegia, varying sensory changes, blindness, hemianopsia, diplopia, vertigo, dysarthria, dysphagia and facial, motor, and sensory change. Investigation includes Doppler ultrasonography of the carotids and vertebrobasilar system. Head CT scan will not show any acute changes. MRA angiography may also be indicated. Selected cases may require endarterectomy, angioplasty or lifelong anticoagulation. Embolic ischemia typically produces symptoms that last longer than three hours and anterior cerebral artery (A) lesions do not produce vertigo and diplopia. Thrombotic stroke (B) indicates infarct, which indicates permanent, not temporary or resolved, neurologic defect. Middle cerebral artery (C) lesions do not produce vertigo.

A 28-year-old woman complains of sudden onset of lower back pain that began two days ago. The pain radiates down both legs. On exam, she has 3/5 strength bilaterally, decreased sensation in her lower extremities, decreased patellar reflexes, and a distended urinary bladder. An MRI of her spine reveals no masses. Which of the following is the most likely diagnosis? Guillain-Barré Syndrome Spinal epidural abscess Syringomyelia Transverse myelitis

Correct Answer ( D ) Explanation: Transverse myelitis is an acute spinal cord disorder characterized by paraplegia, a transverse level of sensory impairment, and sphincter disturbance. The specific etiology is unknown, although it is noted to follow a viral infection in 30% of patients and is commonly referred to as postinfectious myelitis. Patients often complain of back pain and may have a low-grade fever, raising the concern for an epidural abscess. An MRI is usually performed to rule out compressive lesions of the spine. Despite the absence of evidence, most sources still recommend high dose steroids as first line treatment. Although spinal epidural abscess (B) has many clinical similarities to transverse myelitis, the population at risk for this diagnosis is generally limited to individuals with a history of intravenous drug abuse or prior spinal instrumentation. Guillain-Barré syndrome (A) is an acute demyelinating disorder of the peripheral nerves. It is classically associated with an antecedent viral illness followed by the onset of ascending, symmetric paresthesias, and motor weakness. Unlike transverse myelitis, GBS is less commonly associated with back pain and usually occurs over days to weeks whereas transverse myelitis occurs more acutely. Syringomyelia (C) results from a CSF fluid collection within the spinal cord. Any location is possible, but it is most commonly seen in the cervical spine in association with Arnold-Chiari malformation. An MRI will confirm the diagnosis.

10-year-old boy returns to the ED for the second time in a week complaining of a right-sided headache. Five days ago he was also seen for a right-sided headache, thick nasal drainage, and fever. A diagnosis of acute sinusitis was made, and he was discharged on amoxicillin/clavulanate, nasal spray, and decongestants. Today, he returns for continued pain above his right eye, fever, vomiting, and confusion. Vital signs are T 39.4°C, BP 85/40, HR 130, and RR 32. His exam is notable for drowsiness, confusion, yellow nasal discharge from both nares, and tachycardia. Which of the following findings confirms the diagnosis? Elevated C-reactive protein (CRP) and erythrocyte sedimentation rate (ESR) Elevated leukocytes, low protein, and normal glucose in the CSF Elevated white blood cell count with left shift Positive Influenza antigen test Ring-enhancing lesion seen on CT scan of the head with IV contrast

Correct Answer ( E ) Explanation: This child has an intracranial abscess, caused by local spread of infection from his frontal sinus. The imaging modality of choice is CT or MRI with contrast, which may show one or more ring-enhancing lesions. The ring of contrast enhancement is the inflammatory capsule surrounding the abscess. Infection reaches the brain via three mechanisms: (1) contiguous spread from the middle ear, sinus, mastoid, or teeth; (2) directly from trauma or neurosurgery; (3) hematogenously (endocarditis, skin, pelvic, intrabdominal, and pulmonary infections.) The most common symptom is headache, with a third of patients also having fever and a focal neurologic deficit. Other symptoms stem from increased intracranial pressure and include hemiparesis, seizure, vomiting, and confusion Elevated CRP and ESR (A) are also nonspecific markers of inflammation and cannot be used to confirm the diagnosis. CSF with increased leukocytes (B) raises the suspicion for a CNS infection but does not confirm the diagnosis. If there is a high-clinical suspicion that an intracranial abscess is present, then a lumbar puncture should be avoided due to the increased risk of brain herniation. An elevated white blood cell count (C) is nonspecific and cannot confirm the diagnosis. A concomitant Influenza infection (D) is possible but does not explain all of the patient's findings.

A 55-year-old woman without significant past medical history presents to the ED for right-sided facial weakness that began acutely one hour prior to arrival. She woke up from sleep this morning with a dry right eye and is now unable to move the right side of her mouth. You ask the patient to smile and raise her eyebrows and you note a right-sided facial droop and no forehead wrinkling on the right side. Her neurological exam and vital signs are otherwise normal. Which of the following is the most appropriate next step in management? Administer a small dose of edrophonium, followed by a full dose, and observe the patient for improvement of her symptoms Administer intravenous prochlorperazine Obtain a blood glucose level, basic laboratory work, and CT scan of the head Obtain a National Institutes of Health Stroke Scale, bring the patient to the CT scanner, and consult a neurologist Provide prescriptions for corticosteroids, antivirals, artificial tears, and an eye patch, and have the patient follow up with her physician

Correct Answer ( E ) Explanation: This patient has Bell's palsy, a partial or complete paralysis of the facial nerve (cranial nerve VII). Typical symptoms include sudden onset of unilateral facial paralysis, including the forehead, decreased tearing, hyperacusis, and loss of taste sensation on the anterior two-thirds of the tongue that can progress over 1 to 7 days. Many patients describe a viral prodrome. The physical exam is key in the diagnosis to establish the presence of a peripheral rather than a central seventh nerve palsy. Upper and lower facial weakness is present in the peripheral nerve palsy. In central seventh nerve palsy, common in stroke syndromes, only the lower facial weakness is present. The forehead is spared because of bilateral hemispheric innervations to the forehead muscles. Treatment includes corticosteroids, antivirals (for patients with severe to complete paralysis), artificial tears, and eye protection. The National Institutes of Health Stroke Scale (D) helps to classify the severity of deficit in the setting of an acute CVA and should be obtained in all suspected cases. The patient in this vignette, however, had clear evidence of a peripheral seventh nerve palsy, so a stroke scale is not needed. Prochlorperazine (B) is often used in the treatment of migraine headaches but has no role in the management of Bell's palsy. Obtaining a blood glucose level (C) is important to rule out hypoglycemia or hyperglycemia as a cause of neurologic deficit and would be reasonable in this case, but further laboratory or radiographic evaluation is not necessary. Imaging should be considered in patients with an atypical presentation which is not the case with the patient above. Administration of edrophonium (A), a short-acting acetylcholine esterase inhibitor, is useful for suspected cases of myasthenia gravis (the Tensilon challenge test) and can help distinguish exacerbations from cholinergic crises. Although myasthenia gravis often involves facial (especially ocular) muscle weakness, systemic symptoms are not uncommon and, unlike Bell's palsy, deficits tend to worsen with activity.

A 19-year-old woman presents to the ED complaining of a severe headache that started suddenly one hour prior to arrival. The headache is associated with nausea, vomiting, and photophobia. The patient reports no history of headaches. On physical exam, her BP is 160/95 mm Hg, HR 115, RR 24, T 99.1°F, and oxygen saturation 99% on room air. Her pupils are equal, round, and reactive to light. Her optic discs are sharp. Heart, lung, abdominal, and neurologic exams are unremarkable. She is administered IV prochlorperazine. Her symptoms improve but do not resolve. Which of the following is the most appropriate next step in management? Administer empiric antibiotics followed by lumbar puncture Discharge home with outpatient neurology follow-up MRI/MRA of head Noncontrast head CT scan, followed by contrast head CT scan Noncontrast head CT scan, followed by lumbar puncture if CT scan is negative

Correct Answer ( E ) Explanation: This patient has a subarachnoid hemorrhage (SAH). The most common cause of nontraumatic SAH is rupture of an intracranial aneurysm. The most common presenting symptom of SAH is sudden onset of severe headache with maximal intensity within minutes. SAH headache is classically described as a thunderclap headache. The diagnosis of SAH should begin with radiographic analysis. Noncontrast CT remains the initial test of choice, with a sensitivity of 98%-100% in the first 12 hours following SAH. Sensitivity declines with time and falls to 93% at 24 hours and to as low as 57% at six days after the event. Because the diagnostic sensitivity of CT scanning is not 100%, diagnostic lumbar puncture should be performed if the initial CT scan is negative. The spinal fluid is evaluated for xanthochromia and red blood cell count. Xanthochromia is a yellow appearance of the CSF due to blood breakdown and release of bilirubin. Xanthochromia may take up to 12 hours to develop and remain present for several weeks. Tubes one and four should be sent for cell count. There is no accepted cutoff for the number of RBCs that determine a positive LP, but a comparison of RBC counts between tubes one and four can be used to differentiate a possible intracranial bleed (no change between tubes) from a traumatic tap (count should decline). Empiric antibiotic administration (A) and lumbar puncture are appropriate if you have a clinical suspicion for meningitis. SAH and meningitis have many overlapping features. However, meningitis is typically associated with fever and is more indolent in onset. It is critical to remember that improvement of pain in response to medication therapy used for headache control does not rule out SAH (B). This patient should not be sent home. It is estimated that approximately between 5% and 12% of patients with SAH remain undiagnosed. Failure to diagnose a SAH considerably increases morbidity and mortality. MRI and MRA (C) are helpful tools in identifying cerebral aneurysms. Sensitivity for MRA is limited and, much like CTA, best with aneurysms > 5 mm in diameter. Also, many emergency departments do not have access to these modalities. Contrast head CTs (D) are useful in diagnosing abscesses and tumors. Many centers are beginning to use CT angiography to look for aneurysms after a normal noncontrast study. However, sensitivity varies based on size, decreasing significantly for aneurysms < 5 mm in diameter. Consequently, CTA has not yet supplanted lumbar puncture as the definitive test for suspected SAH, but data is evolving.

Rapid Review Idiopathic Intracranial Hypertension (Pseudotumor Cerebri)

Idiopathic Intracranial Hypertension (Pseudotumor Cerebri) Patient will be a young obese female With a history of vitamin A toxicity, use of steroids or tetracyclines Complaining of HA and visual sx PE will show papilledema, CN VI palsy Labs will show opening pressure on LP Treatment is acetazolamide, serial LPs, weight loss


Set pelajaran terkait

General Mathematics - Arithmetic Basics

View Set

Vocabulary: Habitat, Niche, Competition, Predation, Symbiosis, Mutualism, Commensalism, Parasitism

View Set

NCLEX 4000 Questions with answers Health Assessment

View Set

Systems Security 1 Final Exam Review

View Set

Chapter 18 Disinfection & Sterilization

View Set

psychology chapter 4 fill in the blank

View Set

World geography: chapter 28 vocabulary

View Set

Radius, Diameter, Circumference and Area of Circles

View Set